Sei sulla pagina 1di 124

Ministrio da Educao - MEC

Universidade Aberta do Brasil


Instituto Federal de Educao, Cincia e Tecnologia do Cear
Diretoria de Educao a Distncia

LICENCIATURA EM MATEMTICA
Jnio Klo de Sousa Castro

Teoria

dos nmeros
FORTALEZA

2010

Crditos
Presidente

Luiz Incio Lula da Silva

Ministro da Educao
Fernando Haddad

Secretrio da SEED

Carlos Eduardo Bielschowsky

Diretor de Educao a Distncia


Celso Costa

Reitor do IFCE

Cludio Ricardo Gomes de Lima

Pr-Reitor de Ensino
Gilmar Lopes Ribeiro

Diretora de EAD/IFCE e Coordenadora


UAB/IFCE
Cassandra Ribeiro Joye

Vice-Coordenadora UAB
Rgia Talina Silva Arajo

Coordenador do Curso de
Tecnologia em Hotelaria
Jos Solon Sales e Silva

Coordenador do Curso de
Licenciatura em Matemtica
Zelalber Gondim Guimares

Elaborao do contedo

Equipe Arte, Criao e Produo Visual

bner Di Cavalcanti Medeiros


Benghson da Silveira Dantas
Davi Jucimon Monteiro
Diemano Bruno Lima Nbrega
Germano Jos Barros Pinheiro
Gilvandenys Leite Sales Jnior
Hommel Almeida de Barros Lima
Jos Albrio Beserra
Jos Stelio Sampaio Bastos Neto
Larissa Miranda Cunha
Marco Augusto M. Oliveira Jnior
Navar de Medeiros Mendona e Nascimento
Roland Gabriel Nogueira Molina

Equipe Web

Aline Mariana Bispo de Lima


Benghson da Silveira Dantas
Fabrice Marc Joye
Igor Flvio Simes de Sousa
Luiz Bezerra de Andrade FIlho
Lucas do Amaral Saboya
Marcos do Nascimento Portela
Ricardo Werlang
Samantha Onofre Lssio
Tibrio Bezerra Soares
Thuan Saraiva Nabuco

Reviso Textual

Autor: Jnio Klo de Sousa Castro

Aurea Suely Zavam


Nukcia Meyre Arajo de Almeida

Equipe Pedaggica e Design Instrucional

Reviso Web

Ana Cladia Ucha Arajo


Andra Maria Rocha Rodrigues
Cristiane Borges Braga
Eliana Moreira de Oliveira
Gina Maria Porto de Aguiar Vieira
Iraci Moraes Schmidlin
Jane Fontes Guedes
Jivago Silva Arajo
Karine Nascimento Portela
Lvia Maria de Lima Santiago
Luciana Andrade Rodrigues
Maria Irene Silva de Moura
Maria Vanda Silvino da Silva
Marlia Maia Moreira
Regina Santos Young

Dbora Liberato Arruda Hissa


Saulo Garcia

Logstica

Francisco Roberto Dias de Aguiar


Virgnia Ferreira Moreira

Secretrios

Breno Giovanni Silva Arajo


Francisca Venncio da Silva

Auxiliar

Bernardo Matias de Carvalho


Carla Anale Moreira de Oliveira
Maria Tatiana Gomes da Silva
Wagner Souto Fernandes
Zuila Smea Vieira de Arajo

ISBN 978-85-63953-22-3

Catalogao na Fonte: Islnia Fernandes Arajo CRB 3/917


C355t

Castro, Jnio Klo Sousa


Teoria dos nmeros: semestre III. / Jnio Klo Sousa de Castro;
Coordenao Cassandra Ribeiro Joye - Fortaleza: UAB/IFCE, 2010.
124p. : il. ; 27cm.
1. CONGRUNCIA 2. DIVISIBILIDADE 3. NMEROS PRIMOS. I. Joye, Cassandra Ribeiro. (Coord.) II. Instituto Federal de
Educao, Cincia e Tecnologia do Cear - IFCE III. Universidade
Aberta do Brasil UAB. IV. Ttulo
CDD 512.70785

Sumrio
Apresentao

Aula 1 - Divisores de um nmero

Tpico 4 - O Teorema Fundamental da Aritmtica

8
14
20
26

Aula 2 - Mltiplos

31

Tpico 1 - Divisibilidade
Tpico 2 - Nmeros primos
Tpico 3 - Diviso de inteiros e o algoritmo de Euclides

Tpico 3 - Congruncia
Tpico 4 - Critrios de divisibilidade

32
34
38
42

Aula 3 - Alguns teoremas sobre Congruncia

47

Tpico 1 - Teorema de Wilson


Tpico 2 - Teorema de Fermat

48
54
56

Tpico 1 - Mnimo Mltiplo Comum


Tpico 2 - Outras bases

Tpico 3 - Teorema de Euler

Aula 4 - Funes aritmticas - parte I


Tpico 1 - As funes t e s

61

Tpico 3 - A funo m de Mbius

62
68
72

Aula 5 - Funes aritmticas - parte II

75

Tpico 2 - A funo f de Euler

Tpico 3 - Outras relaes

76
80
84

Aula 6 - O princpio das gavetas

87

Tpico 1 - Introduo

88
92
96

Tpico 1 - Outras propriedades das funes aritmticas


Tpico 2 - A funo maior inteiro

Tpico 2 - Generalizao do princpio das gavetas


Tpico 3 - Exemplos gerais

Aula 7 - Resduos quadrticos


Tpico 1 - Resduos quadrticos
Tpico 2 - O Smbolo de Legendre
Tpico 3 - Lei da reciprocidade quadrtica

99
100
106
110

Aula 8 - Problemas diversos

115

Tpico 1 - Miscelnea de exerccios

116

Referncias

123

Currculo

124

Apresentao
Caro (a) aluno (a), no texto que segue temos a apresentao de algumas propriedades aritmticas dos nmeros inteiros, especialmente aquelas referentes ao algoritmo da diviso.
Como pr-requisito para a sua leitura, recomenda-se alguma familiaridade com as operaes aritmticas fundamentais - adio e multiplicao.
Os dois primeiros captulos tratam dos divisores e dos mltiplos de um nmero inteiro,
com enfoque nos nmeros primos, e apresentamos a noo de congruncia, aprofundada
no terceiro captulo.
As funes aritmticas m, s, t e f so estudadas nos dois captulos seguintes, nos quais h
exemplos computacionais e relaes entre elas.
O princpio de Dirichlet (ou das gavetas) abordado no sexto captulo.
No penltimo captulo, estudamos os resduos quadrticos, com nfase no uso do smbolo
de Legendre e da lei da reciprocidade quadrtica, com a qual encerramos a teoria contida
neste material.
O ltimo captulo apresenta uma miscelnea de exerccios sobre os diversos assuntos
abordados. De posse deste livro, este ltimo captulo pode (e deve) ser consultado a qualquer momento, para melhor fixao da teoria.
Desejando a todos um bom proveito na leitura e um bom aprendizado, s resta comear
o trabalho.
Jnio Klo

Aula 1

Divisores de um nmero
Ol, a todos.

Em nossa primeira aula de Teoria dos Nmeros, estudaremos o processo de diviso


de nmeros inteiros, detalhando e justificando suas principais propriedades. Alguns
dos assuntos so nossos conhecidos de longa data, pois trataremos do conjunto
e das operaes de soma e multiplicao. Alm de acompanhar os exemplos fornecidos neste texto, no hesite em fazer testes para a verificao das propriedades e
melhor assimilao das definies.

Objetivos da aula:
Definir os principais termos da Teoria dos Nmeros.
Analisar a diviso de nmeros inteiros e os algoritmos correlatos.

01

RVORES
DIVISIBILIDADE
DE
POSSIBILIDADES

TPICO

OBJETIVOS
Identificar as principais definies sobre os nmeros
inteiros e suas consequncias.
Estabelecer o conceito de divisibilidade e as relaes
entre os divisores de um nmero inteiro.

Teoria dos Nmeros, do ponto de vista clssico, trata principalmente


do conjunto dos nmeros inteiros, denotado por , que compreende
todos os nmeros naturais positivos, o zero e seus simtricos.

= {...,-2,-1,0,1,2,3,...}

Para dois nmeros inteiros a e b ,


so definidas

as operaes de soma, represen


tada infixamente por a + b , e de multiplicao (ou produto), representada por a.b
ou simplesmente ab , que satisfazem as seguintes propriedades:
1. "a, b , vale a + b = b + a

(a soma comutativa)

2. "a, b, c , vale (a + b) + c = a + (b + c )

(a soma associativa)

3. "a , vale a + 0 = a

(existe um elemento
neutro para a soma)

4. "a , $(-a ) , tal que a + (-a ) = 0

(todo elemento possui


inverso para a soma)

Licenciatura em Matemtica

(a multiplicao

5. "a, b , vale ab = ba

comutativa)
(a multiplicao

6. "a, b, c , vale (ab)c = a(bc )

associativa)
7. "a , vale a.1 = a

(existe um elemento neu

A
1

tro para a multiplicao)


8. "a, b, c , vale a(b + c ) = ab + ac

(a multiplicao distri

T
1

butiva em relao soma)


Por causa da associatividade, podemos fazer a soma de
qualquer quantidade finita de inteiros agrupando-os em qual
quer ordem e, assim, os parnteses sero opcionais nesse ca
so, bem como no produto de uma sequncia finita de inteiros.
Aqui temos a relao de ordem:
... < -2 < -1 < 0 < 1 < 2 < 3 < ... , na qual entre dois nme
ros listados consecutivamente no h nenhum nmero inteiro.
relevante observar que no limitado superiormente,
ou seja, no possui um elemento mximo, bem como no
limitado inferiormente, pois no possui um elemento mnimo.
Alm disso, se a < b , ento a + c < b + c , "c e
ac < bc , "c > 0 .

ATENO!
Observao: A um conjunto
com operaes de soma e
multiplicao que satisfazem as
propriedades (1), (2), (3), (4), (6)
e (8) damos o nome de anel; caso
a propriedade (5) seja satisfeita,
o anel dito comutativo. Se (7)
satisfeita, dizemos que o anel
possui identidade. O conjunto dos
nmeros inteiros , portanto, um
anel comutativo com identidade.

EXEMPLO 1

Mostre que no existe o inverso multiplicativo do nmero 2.


Soluo: Suponha que exista a tal que 2.a = 1 . Por
um lado, este nmero deve ser maior que 0, pois, se a 0 , te
ramos 1 = 2a 2.0 = 0 , ou seja, 1 0 , que falso. Da mesma
forma, se a 1 , teramos 1 = 2a 2.1 = 2 , ou seja, 1 2 , que
falso, de onde conclumos que a < 1 . Assim, deveramos ter
a > 0 e a < 1 , mas sabemos que tal inteiro no existe. Logo 2
no possui inverso multiplicativo.
O mesmo raciocnio do exemplo acima pode ser aplicado
para mostrar que os nicos nmeros inteiros que possuem in
verso multiplicativo em so 1 e 1. A estes nmeros damos
o nome de unidades e denotaremos por U () . Assim defini

ATENO!
Embora saibamos de antemo
que o inverso do nmero 2 o
nmero 1/2, que no inteiro,
esse conhecimento prvio no
deve ser tomado como imediato,
pois envolve, na maioria dos
casos, teorias mais elaboradas,
como o estudo dos nmeros
racionais. Devemos, assim, tomar
cuidado com o que parecer bvio
e tentar provar usando apenas as
propriedades de cada objeto que
estivermos analisando.

Teoria dos nmeros

mos U () = {n ; $m , mn = 1} e vale U () = {-1,1} .


Passemos definio central desta aula.
Definio 1: Dados os nmeros inteiros a e b , dizemos que a divide b (repre
sentamos por a |b ) se existir um inteiro n tal que b = a.n . Ou seja:
a |b $n ; b = an .

A
1

Quando a |b , dizemos tambm que a um divisor de b ou, equivalentemente,


que b um mltiplo de a .

T
1

EXEMPLO 2A

Como 30 = 5.6, podemos dizer que 5|30 e 6|30 , ou seja, 5 e 6 so divisores de 30.
EXEMPLO 2B

Podemos usar uma ideia semelhante do exemplo 1 para mostrar que no existe
nenhum inteiro n para o qual 5n = 12 (tente repetir o processo de modo a demons
trar isso). Assim sendo, 5 no divide 12, o que pode ser representado por 5|12 , isto
, 12 no um mltiplo de 5.
Proposio 1: A divisibilidade uma relao transitiva, ou seja, se a |b e b|c ,
ento a |c .
Demonstrao: Se a |b , ento b = an1 . Se b|c , ento c = bn2 = an1n2 , logo a |c .
Proposio 2: Se a |b e b|a , ento a = b ou a = -b .
Demonstrao: Se a |b , ento b = an1 . Se b|a , ento a = bn2 = an1n2 , logo
n1n2 = 1 , o que somente ocorre se n1 = n2 = 1 , caso em que a = b , ou n1 = n2 = -1 ,
caso em que temos a = -b .
Proposio 3: Se a |b e c , com c 0 , ento ac |bc .
Demonstrao: Se a |b , ento b = an . Multiplicando esta igualdade por c ,
temos bc = acn , logo ac |bc .
Proposio 4: Se a |b e a |c , ento a |b + c .
Demonstrao: Se a |b e a |c , ento existem inteiros m e n para os quais
b = am e c = an . Assim, b + c = am + an = a(m + n) , logo a |b + c .
Observao 1: Como recurso extra para o entendimento da expresso a |b + c ,

10

Licenciatura em Matemtica

poderamos usar parnteses e escrever a |(b + c ) , entretanto a ausncia deles no gera


nenhuma ambiguidade, pois a expresso (a |b) + c no tem sentido definido.
Observao 2: Como consequncia direta das duas ltimas proposies, se a |b
e a |c , ento a |mb + nc para quaisquer inteiros m e n .
A respeito dos divisores de um nmero, valem tambm as seguintes propriedades, que
so consequncias diretas da definio e cujas demonstraes so sugeridas como exerccio.

A
1

1) 1|a , a |a e a |0 , para qualquer inteiro a ;


2) se ab|ac e a 0 , ento b|c ;

T
1

3) se a |b , ento a |-b , -a |b e -a |-b ;


4) se a |b e b 0 , ento a b ;
b
5) se a |b , ento c = inteiro e c |b .
a
Uma das implicaes da propriedade 4 que o conjunto de divisores inteiros de
um nmero no nulo limitado e, por isso, finito. Como resultado da propriedade 3,
a quantidade de divisores inteiros de um nmero no nulo sempre par, j que sem
pre viro aos pares cada divisor e seu simtrico. Assim, basta conhecermos apenas
os divisores inteiros positivos de um nmero, pois os negativos estaro automatica
mente determinados. Denotaremos, ento, por D(n) o conjunto de divisores inteiros
positivos do nmero n , ou seja, D(n) = {m + ; m |n} .
EXEMPLO 3

Fazendo
vos

de

testes
a

8,

com
vemos

os
que

positi

ATENO!

D(8) = {1,2,4,8} .

Pela transitividade da relao


de divisibilidade, verifica-se
facilmente que se a | b , ento
D (a ) D (b ) .

inteiros

Assim, os divisores inteiros de 8 so 1 , 2 , 4 e 8 .

Definio 2: O nmero inteiro n par se 2|n e mpar,


caso contrrio.
Durante nossa primeira aula, estudaremos mtodos que, entre outras coisas, nos
fornecero a quantidade de divisores positivos de um nmero inteiro. Antes dis
so, observe o seguinte exemplo prtico de como construir o conjunto D(n) , usando
principalmente a propriedade 5, indicada acima.
EXEMPLO 4A

Escreva o conjunto de divisores positivos do nmero 60.

Teoria dos nmeros

11

Soluo:

De princpio, sabemos que 1 e 60 devem entrar na lista, pois cada nmero inteiro
tem pelo menos dois divisores positivos. Uma vez que podemos escrever 60 = 2.30,
podemos afirmar que 2|60 (60 par) e, pelo mesmo motivo, 30|60 . Como no h
nenhum inteiro entre 1 e 2, no h nenhum divisor inteiro de 60 entre 30 e 60. Assim,
os demais divisores esto entre 2 e 30. Fazendo testes semelhantes aos do primeiro

A
1

exemplo, conclumos que 60 = 3.20. Assim, 3 D(60) e 20 D(60) , e no h nme


ros entre 20 e 30 a serem considerados. Continuando assim entre 3 e 20, o prximo
divisor de 60 o nmero 4, pois 60 = 4.15. Entre 4 e 15, o prximo divisor de 60 o

T
1

nmero 5, pois 60 = 5.12. Entre 5 e 12, o prximo divisor de 60 6, pois 60 = 6.10.


Por fim, testando os nmeros entre 6 e 10, no encontramos nenhum divisor de 60 e
encerramos a lista. D(60) = {1,2,3,4,5,6,10,12,15,20,30,60} .
EXEMPLO 4B

Realizando processo semelhante, podemos concluir que D(28) = {1,2,4,7,14,28}


e D(11) = {1,11} .
Observao 3: Se os nmeros inteiros positivos a e b so tais que a |b , com
a 1 e a b , dizemos que a um divisor prprio de b . Assim, no exemplo anterior,
os divisores prprios de 60 so 2, 3, 4, 5, 6, 10, 12, 15, 20 e 30, e o nmero 11 no tem
divisores prprios.
Observao 4: Para qualquer inteiro positivo n , o teste com os divisores s
necessrio para os nmeros menores ou iguais a

n por causa da propriedade 5.

Uma vez que sabemos construir o conjunto dos divisores inteiros positivos de
um nmero, ainda mais simples construir o conjunto dos divisores de um dos seus
divisores. Do que obtivemos no exemplo 4a, ficaria bem simples encontrar os divi
sores de 30 ou de 15. Pelo resto de nossa aula, teremos a relao de divisibilidade
como objeto relevante, acrescentando novas ferramentas e aprofundando com con
sequncias interessantes.

12

Licenciatura em Matemtica

02
TPICO

NMEROS PRIMOS

OBJETIVOS
Definir nmero primo e estudar suas propriedades;
Descrever o crivo de Erasttenes.

uando analisamos os divisores de um nmero, encontramos maneiras de


fator-los, ou seja, de escrev-los como produto de outros nmeros, por
exemplo escrevemos 60 da forma 2.30. Obviamente, podemos tambm fa

zer 60 = 1.60, que pode ser chamada de fatorao trivial. Neste tpico, estudaremos
especificamente os nmeros que no podem ser fatorados de maneira no trivial que
no podem ser escritos como produtos de fatores menores. Comearemos com a de
finio central a seguir:
Definio 3: Um nmero inteiro p > 1 dito primo se sempre que p|ab obti
vermos p|a ou p|b .

14

Licenciatura em Matemtica

Guarde bem esta definio, pois ela ser revisitada em outros cursos, como o de
Estruturas Algbricas, nos quais se estudam outros conjuntos dentro dos quais a
ideia de elemento primo tambm relevante. Para nmeros inteiros, podemos traba
lhar com uma definio equivalente, como a que segue.
Suponha que o nmero inteiro positivo d seja um divisor do nmero primo p > 1 ,
ou seja, que p = dn , para algum inteiro positivo n . Assim, os nmeros a = d 2 e

A
1

b = n2 so tais que ab = d 2n2 = (dn)2 = (dn)(dn) = p. p , logo p|ab . Pela definio


de nmero primo, temos p|a ou p|b . Uma vez que p|a equivale a p|d 2 , significa
que p|d , mas, como d | p e so ambos positivos, conclumos que d = p e n = 1 . A

T
2

outra alternativa seria p|b , que equivale a p|n2 , logo p|n , mas, como n| p , con
clumos que n = p e d = 1 . Aqui demonstramos que, se primo, um nmero possui
exatamente dois divisores positivos. A recproca
dessa afirmao verdadeira e sua demonstrao
deixada como exerccio.
Assim, obtemos que p > 1 primo se, e
somente se D( p) = {1, p} , ou seja, um nmero
maior que 1 primo quando no possui diviso
res prprios ou ainda quando possui exatamen

GUARDE BEM ISSO!


1. Se o nmero inteiro positivo n > 1 no
primo, dizemos que ele composto, pois
ele pode ser escrito como produto de dois
nmeros menores que ele.

te dois divisores positivos.

2. Todo nmero composto tem pelo menos


um nmero primo como divisor (veremos a
demonstrao ainda nesta aula).

EXEMPLO 5

3. Os nmeros 0 e 1 no so primos nem


compostos, por definio.

Analisando os dados do exemplo 4, vemos que


60 no primo, pois o conjunto de seus divisores

prprios no vazio. O nmero 28 mltiplo de 2, logo no primo, enquanto 11 pos


sui exatamente 2 divisores positivos, sendo, portanto, primo.
Os nmeros primos funcionam como os tomos dos nmeros inteiros positi
vos, pois, como veremos adiante, todo nmero inteiro positivo pode ser escrito
como produto de nmeros primos e, igualmente importante, essa decomposio
feita de maneira nica.
Como a quantidade de divisores de um nmero inteiro positivo finita, podera
mos nos perguntar qual o maior nmero primo que existe. Uma investigao mais apu
rada nos levaria a uma resposta interessante: no h um maior nmero primo! Acom
panhe o seguinte raciocnio: se houvesse um maior nmero primo, isso significaria
que a quantidade deles finita. Seja, ento, o conjunto P = { p1 , p2 ,..., pk } de to
dos os nmeros primos. Dessa forma, considere o nmero inteiro n = p1 p2 ... pk + 1 ,

Teoria dos nmeros

15

que maior que qualquer elemento de P , logo n P . Alm disso, vemos que pi |n ,
para todo i = 1,..., k , de onde obtemos que n no possuiria nenhum divisor primo,
o que uma contradio, visto que n > 1 . Assim, a suposio de que h uma quan
tidade finita de nmeros primos incorreta. Podemos, ento, enunciar o resultado:
Proposio 5: Existem infinitos nmeros primos.

AA
11

Se testarmos alguns dos nmeros inteiros positivos maiores que 1 para sabermos
seus divisores e determinarmos se eles so primos, podemos concluir que os primei

TT
22

ros cinco nmeros primos so 2, 3, 5, 7 e 11. J que a quantidade de nmeros primos


infinita, poderamos investigar como eles esto distribudos ou quantos deles so
menores que um nmero fixado.
H um algoritmo, conhecido como crivo de Eratstenes (matemtico e gegrafo
grego nascido no sculo III a.C.), que lista os nmeros primos menores que n . Pelo
observado anteriormente, os divisores devem ser procurados apenas at (no mxi
mo)

n.

Observe como funciona o crivo de Erasttenes para n = 60 . Inicialmente, veja


mos que

60 @ 7,7 . Assim, o processo de busca de divisores prprios dos nmeros

da lista ser encerrado no 7. Comecemos listando os nmeros inteiros positivos de 1


a 60, e riscamos o 1, que no primo. O primeiro nmero no marcado o 2, que
primo, destaquemos por colchetes.
1
11
21
31
41
51

[2]
12
22
32
42
52

3
13
23
33
43
53

4
14
24
34
44
54

5
15
25
35
45
55

6
16
26
36
46
56

7
17
27
37
47
57

8
18
28
38
48
58

9
19
29
39
49
59

10
20
30
40
50
60

Em seguida, eliminamos todos os mltiplos seguintes de 2, pois eles no so primos.


1 [2]
11 12
21 22
31 32
41 42
51 52

3
13
23
33
43
53

4
14
24
34
44
54

5
15
25
35
45
55

6
16
26
36
46
56

7
17
27
37
47
57

8
18
28
38
48
58

9
19
29
39
49
59

10
20
30
40
50
60

O primeiro nmero no marcado foi o 3 e como ele no tem divisores primos me


nores que ele, ele primo. Em seguida, eliminamos os mltiplos de 3 (basta contar
de trs em trs).

16

Licenciatura em Matemtica

1
11
21
31
41
51

[2]

[3]

12
22
32
42
52

13
23
33
43
53

4
14
24
34
44
54

5
15
25
35
45
55

6
16
26
36
46
56

7
17
27
37
47
57

8
18
28
38
48
58

9
19
29
39
49
59

10
20
30
40
50
60

A
1

O primeiro nmero no marcado foi o 5 e como ele no tem divisores primos

menores que ele, ele primo. Em seguida, eliminamos os mltiplos de 5.


1 [ 2 ] [ 3 ] 4 [5] 6
7
8
9 10
11 12 13 14 15 16 17 18 19 20

21 22 23 24 25 26 27 28 29 30
31 32 33 34 35 36 37 38 39 40
41 42 43 44 45 46 47 48 49 50
51 52 53 54 55 56 57 58 59 60

T
2

O primeiro nmero no marcado foi o 7 e como ele no tem divisores primos me


nores que ele, ele primo. Em seguida, eliminamos os mltiplos de 7. E como o
ltimo do teste, os que sobrarem sem marcao, so nmeros primos.
1
[2] [3] 4
[11] 12 [13] 14
21 22 [23] 24
[31] 32 33 34
[ 41] 42 [ 43] 44
51 52 [53] 54

[5]
15
25
35
45
55

6
16
26
36
46
56

[7 ] 8
[17 ] 18
27
[37 ]
[ 47 ]
57

28
38
48
58

9
10
[19 ] 20
[29 ] 30
39 40
49 50
[59 ] 60

Assim, os nmeros primos menores que 60 so 2, 3, 5, 7, 11, 13, 17, 19, 23, 29,
31, 37, 41, 43, 47, 53 e 59.

O crivo de Eratstenes pode ser usado para qualquer valor de n , obviamen

te exigindo trabalho crescente. Muitos trabalhos atuais so


feitos com testes de primalidade, alguns demandando esfor
os computacionais muito grandes. Mquinas modernas tra

SAIBA MAIS!

balham neste intuito e o maior nmero primo conhecido


243112698 - 1 , que em notao decimal tem quase treze mil alga

rismos e foi descoberto em 2008.


EXEMPLO 6

Mostre que a equao n3 - 4n - 59 = 0 no possui razes

No site http://fisicomaluco.com/
experimentos/eratostenes/ voc
pode conhecer um pouco mais
sobre Eratstenes e algumas de
suas obras.

inteiras.

Teoria dos nmeros

17

Soluo: A equao dada equivalente a n3 - 4n = 59 , ou ainda n2 (n - 4) = 59 .


Assim n2 |59 . Como 59 primo, seus nicos divisores positivos so 1 e 59. Como
n2 59 , para todo n inteiro, a alternativa seria n2 = 1 , que resulta em n = 1 .
En
tretanto, por substituio direta, vemos que nenhum desses valores raiz da equa
o dada. Assim, ela no possui razes inteiras.

A
1

Vale ressaltar que 2 o nico nmero primo par, pois todos os demais nmeros
pares tm pelo menos o 2 como divisor prprio. Assim, todo nmero primo maior

T
2

que 2 mpar, mas nem todo nmero mpar primo, como bem ilustra o nmero 9.
Conhecer os nmeros primos e suas propriedades , de certa forma, conhecer
todos os nmeros inteiros. Dessa forma, razovel que conheamos pelo menos os
primeiros nmeros primos de cor. A tabela obtida no exemplo do crivo de Erat
stenes um bom modo de fixar essas ideias. Entretanto, como parte fundamental
da teoria dos nmeros inteiros, h muito ainda o que se ver sobre os nmeros pri
mos, e isso se dar nos prximos tpicos.

18

Licenciatura em Matemtica

03
TPICO

DIVISO DE INTEIROS E O
ALGORITMO DE EUCLIDES

OBJETIVOS
Definir quociente e resto na diviso de inteiros;
Destacar as propriedades do mximo divisor comum;
Estudar mtodos de inferncia sobre o conjunto
soluo das equaes diofantinas.

os tpicos anteriores, vimos como determinar se a |b , ou seja, se


existe c tal que b = ac . Nesse caso dizemos que b um mlti
plo de a . Agora veremos o algoritmo da diviso, suas consequn

cias e principais propriedades. Inicialmente, veja que, da mesma forma que de


finimos o conjunto dos divisores positivos de um nmero, podemos definir o
conjunto dos mltiplos positivos de um nmero, o qual denotaremos por M (n) ,
ou seja, M (n) ={m *+ ; n|m} .
De imediato verificamos que os mltiplos positivos do nmero inteiro positivo n
so 1.n , 2.n , 3.n , 4.n , ... Logo, conclumos simplesmente que so infinitos. Entre
tanto, fixado o nmero inteiro a > 0 , o conjunto {m M (n); m a} {0} , dos ml
tiplos no-negativos de n que so menores ou iguais a a , limitado superiormente
e no-vazio, e assume, portanto, um mximo. Seja m0 este mximo. Como m0 um

20

Licenciatura em Matemtica

mltiplo de n , podemos escrever m0 = qn , para algum inteiro positivo q . Necessa


riamente vlido que a - m0 < n , pois, do contrrio, perderamos a maximalidade
de m0 . Se escrevermos r = a - qn , podemos, ento, enunciar o seguinte resultado:
Dados os nmeros inteiros positivos a e n , existem nmeros inteiros q e r , cha
mados respectivamente de quociente e resto da diviso, tais que

A
1

a = qn + r e 0 r < n .

T
3

Assim, dividir a (chamado de dividendo) por n (o divisor) consiste em encon


trar o quociente e o resto que satisfazem a propriedade desejada. Aqui usamos o
artigo definido porque tanto quociente quanto resto so unicamente determinados,
como provado a seguir.
Proposio 6: O quociente e o resto da diviso entre os inteiros positivos a e n
so nicos.
Demonstrao: Suponha que haja inteiros
a = q1n + r1
de, fazemos

a = q2n + r2

0 r1 , r2 < n .

q1 , q2
Dessa

r2 < n r2 - r1 < n - r1 n , ou seja,

r1 , r2

ltima

tais que
desigualda

r2 - r1 < n . Analogamen

te, pode-se demonstrar que -n < r2 - r1 . Das igualdades acima, podemos inferir
q1n + r1 = q2n + r2 (q1 - q2 )n = r2 - r1 , com base na qual podemos afirmar que
n|r2 - r1 . Mas como -n < r2 - r1 < n , a nica possibilidade r2 - r1 = 0 , isto , o
resto nico e, assim, o quociente tambm nico, como se pode verificar da igual
dade (q1 - q2 )n = r2 - r1 = 0 .
EXEMPLO 1A

Uma vez que podemos escrever 13 = 5.2 + 3 e 3 < 5, podemos dizer que a diviso
de 13 por 5 apresenta quociente 2 e resto 3.
EXEMPLO 1B

verdade que 27 = 4.5 + 7, mas no podemos dizer que a diviso de 27 por 4


gera quociente 5 e resto 7, pois 7 no menor que 4. Da mesma maneira, embora
27 = 4.7 + (1), o resto no pode por ser negativo. Na diviso de 27 por 4, o quo
ciente vale 6 e o resto vale 3.
EXEMPLO 1C

Como 30 = 6.5, o quociente e o resto da diviso de 30 por 6 valem 5 e 0, respec


tivamente.

Teoria dos nmeros

21

Observao: Embora a definio acima tenha sido estabelecida para nmeros in


teiros positivos, o processo pode ser, com pequenas adaptaes, estendido para n
meros inteiros quaisquer desde que o divisor seja diferente de zero. Quando o divi
sor for negativo, exigiremos para o resto que ele seja menor que o valor absoluto do
divisor. Assim, fazendo a diviso de 33 por 5, o quociente 7 e o resto 2, pois
33 = (5).(7) + 2 e 0 2 < -5 .

A
1

Pelo que definimos anteriormente, imediato que, quando a |b , o resto da divi


so de b por a 0, caso em que dizemos que a diviso exata.

T
3

Definio 4: Dados os nmeros inteiros positivos a e b , dizemos que o nmero


d + o mximo divisor comum entre a e b , e escrevemos d = (a, b) , quando:
(i) d |a e d |b , ou seja, d D(a ) D(b) ;
(ii) se d1 |a e d1 |b , ento d1 |d , ou seja, d o maior nmero com a propriedade (i).
Quando (a, b) = 1 , dizemos que a e b so relativamente primos ou primos entre si.
EXEMPLO 2A

Como D(60) = {1,2,3,4,5,6,10,12,15,20,30,60} e D(28) = {1,2,4,7,14,28}, temos


D(60) D(28) = {1,2,4} e, assim, (60,28) = 4 .
EXEMPLO 2B

Como D(16) = {1,2,4,8,16} e D(27) = {1,3,9,27} , temos (16,27) = 1 e, assim,


16 e 27 so relativamente primos, embora nenhum deles seja um nmero primo.
Vale ressaltar que, como o nmero 1 divisor de qualquer inteiro, o conjunto dos
divisores comuns a dois inteiros positivos nunca vazio e limitado por a e b , logo
possui um elemento mximo. Podemos, assim, concluir que sempre existe (a, b) .
Ve
jamos, ento, como determinar o mximo divisor comum entre dois nmeros inteiros
sem ter que listar todos os divisores de ambos.
Decorre diretamente da definio que se a |b , ento (a, b) = a e no h o que fazer.
Caso isso no acontea, considere a > b , divida a por b e obtenha um resto r .
Proposio 7: Se r o resto da diviso de a por b , ento (a, b) = (b, r ) .
Demonstrao: Fazendo d = (a, b) , temos que d divisor de a e de b , lo
go podemos escrever a = dq1 e b = dq2 . Como r o resto da diviso de a por b ,
podemos escrever a = bq + r , ou seja, r = a - bq = dq1 - dq2q = d (q1 - qq2 ) . Desse
modo, temos que d um divisor de r .
Agora considere d0 = (b, r ) . Como d divisor de b e de r , temos, pela condio
(ii) da definio, que d divisor de d0 . Se mostrarmos que que d0 divisor de d ,

22

Licenciatura em Matemtica

teremos a igualdade que completa a prova.


Como d0 divisor de b e de r , podemos escrever b = d0 q3 e r = d0 q4 e, fazen
do as devidas substituies, temos: a = bq + r = d0 q3q + d0 q4 = d0 (q3q + q4 ) . As
sim, d0 divisor de a e, como j era divisor de b , temos que d0 divisor de d .
Dessa forma, d divisor de d0 , e d0 divisor de d . Chegamos, ento, concluso
de que so iguais, ou seja (a, b) = (b, r ) .

A
1

A proposio anterior apenas transfere o problema de determinar o mximo di


visor comum entre os nmeros a e b para encontrar o mximo divisor comum en

T
3

tre b e r , o resto da diviso de a por b , mas com a vantagem de que os nmeros


envolvidos so menores. Se r |b , vale dizer que (b, r ) = r , e o processo, ento, se
encerra. Caso contrrio, repetimos este passo: se r2 for o resto da diviso de b por
r , temos (b, r ) = (r , r2 ) , e assim sucessivamente at que encontremos uma diviso
exata, caso em que o divisor ser o mximo divisor comum entre os nmeros iniciais.
O processo descrito conhecido como Algoritmo de Euclides.
EXEMPLO 3A

Determine o mximo divisor entre 60 e 28.


Soluo:

Comecemos dividindo 60 por 28; obtemos quociente 2 e resto 4. Assim


(60,28) = (28,4) e como 4|28 , vale (28,4) = 4 , ou seja (60,28) = 4 .
EXEMPLO 3B

Determine o mximo divisor comum entre 129 e 45.


Soluo:

Observe que 129 = 45.2 + 39, assim transferimos o problema para os nmeros 45
e 39. Mas 45 = 39.1 + 6, e o problema passa para 39 e 6. Temos 39 = 6.6 + 3. Por fim,
o problema de encontrar o mximo divisor comum entre 3 e 6 resolve-se diretamente
do fato de 3 ser um divisor de 6, de modo que 3 o mximo divisor comum entre 3
e 6 e, logo, entre 129 e 45.
EXEMPLO 3C

Determine o mximo divisor comum entre 400 e 148.


Soluo:

400 = 2.148 + 104


148 = 1.104 + 44
104 = 2.44 + 16
44 = 2.16 + 12

(400, 148) = (148, 104)


(148, 104) = (104, 44)
(104, 44) = (44, 16)
(44, 16) = (16, 12)

Teoria dos nmeros

23

16 = 1.12 + 4 (16, 12) = (12, 4)


12 = 3.4 + 0.

(12, 4) = 4

Assim, (400, 148) = 4.


Decorre tambm do algoritmo de Euclides que, se d = (a, b) , ento existem
m, n , tais que d = ma + nb . Adiante ser provado que d o menor inteiro posi

tivo que pode ser escrito dessa forma. Assim, por exemplo, a equao 30m + 15n = 1

A
1

no tem razes inteiras porque 30 e 15 no so relativamente primos. Podemos dizer

T
3

EXEMPLO 4A

que 1 no pode ser escrito como combinao linear de 30 e 15.

Encontre m, n tais que 129m + 45n = 3 .


Soluo:

Uma vez que (129, 45) = 3, o problema possvel. Pelo algoritmo de Euclides,
podemos proceder:
129 = 2.45 + 39, daqui podemos dizer que 39 = 129 2.45
45 = 1.39 + 6, assim 6 = 45 39 = 45 (129 2.45) = 3.45 129
39 = 6.6 + 3.
Logo
3 = 39 6.6 = (129 2.45) 6.(3.45 129) = 129 2.45 18.45 + 6.129 = 7.129 20.45.
Assim, os valores m = 7 e n = -20 satisfazem a relao 129m + 45n = 3 .
Observe que podemos encontrar solues inteiras para 129m + 45n = d para
qualquer inteiro d mltiplo de 3, bastando para isso multiplicar as solues da
equao original.
EXEMPLO 4B

Encontre dois nmeros inteiros a, b tais que 60a + 25b = 30 .


Soluo:

Para que o algoritmo de Euclides fornea uma soluo para o problema, neces
srio que 30 seja mltiplo do mximo divisor comum entre 60 e 25. Comecemos de
terminando (60, 25).
60 = 2.25 + 10
25 = 2.10 + 5
20 = 4.5.
Assim, encontramos (60, 25) = 5 e, aplicando o mtodo descrito no exemplo an
terior, podemos encontrar 5 como combinao linear de 60 e 25. O processo leva
igualdade

24

Licenciatura em Matemtica

5 = (2).60 + 5.25
Como 30 = 6.5, basta multiplicar a ltima igualdade por 6 para obter:
6.5 = 6.(2).60 + 6.5.25
E assim conclumos que 30 = (12).60 + 30.25, ou seja, os valores a = -12 e
b = 30 satisfazem a relao 60a + 25b = 30 .

A
1

Definio 5: Dados os nmeros inteiros a, b, c , a equao ax + by = c com in


cgnitas x e y chamada de equao diofantina linear (em referncia a Diofante,
matemtico e gegrafo considerado por muitos o maior algebrista grego, o qual tem

T
3

para a Aritmtica a importncia que Euclides tem para a Geometria).



Proposio 8: A equao diofantina linear de duas incgnitas x e y dada por
ax + by = c admite soluo se, e somente se, (a, b)|c .
Demonstrao: Suponha que a equao ax + by = c tenha uma soluo,
ou seja, que existam inteiros x1 e y1 tais que ax1 + by1 = c . Sendo d = (a, b) ,
podemos escrever

a = dq1

b = dq2 , para inteiros apropriados. Assim

c = dq1x1 + dq2 y1 = d (q1x1 + q2 y1 ) . Conclumos, ento, que d |c .


Pelo algoritmo de Euclides, podemos escrever d = ax1 + by1 para inteiros apro
priados, mas, se supusermos que d |c , poderemos escrever c = dq , onde q seria
um inteiro. Assim c = dq = q(ax1 + by1 ) = (qx1 )a + (qy1 )b . Logo,

obtemos que os in
teiros x = qx1 e y = qy1 satisfazem a relao ax + by = c e, assim, a equao tem
soluo inteira, o que completa a prova.
O algoritmo de Euclides para a determinao do mximo divisor comum entre dois
nmeros fornece, como visto, uma maneira de resolver as equaes diofantinas. Veri
fique nos exemplos que antecedem a definio de equao diofantina que poderamos
ter feito um teste com o mximo divisor comum entre os coeficientes para determinar
se a equao era possvel. Revise os conceitos da aula e faa testes com dois nmeros a
quaisquer para treinar a tcnica. Em seguida, podemos passar ao prximo tpico, que
trata do Teorema Fundamental da Aritmtica e de suas implicaes.

Teoria dos nmeros

25

04
TPICO

O TEOREMA FUNDAMENTAL
DA ARITMTICA

OBJETIVOS
Enunciar e demonstrar um teorema central para o
curso;
Observar as principais consequncias do teorema e
suas aplicaes.

imos que um nmero primo possui exatamente dois divisores positivos


e, portanto, no pode ser escrito como produto de dois nmeros menores
que ele. Assim, como 29 um nmero primo, a nica forma de escrev-lo

como produto de dois nmeros inteiros positivos 29 = 1.29.


Os nmeros compostos possuem divisores triviais, podendo ser fatorados como
produto de nmeros menores. Por exemplo, 30 composto e pode ser escrito como
30 = 6.5 = 2.15 = 3.10. Algo a se observar aqui que nenhuma dessas maneiras de
escrever o nmero 30 envolve apenas primos. De fato, a nica maneira (a menos da
ordem dos nmeros) de escrever 30 como produto de nmeros primos 30 = 2.3.5.
A seguir, veremos que podemos fazer isso com qualquer nmero inteiro maior que 1.
Comecemos por um resultado simples que servir para, de certa forma, tornar a
demonstrao do teorema mais clara.

26

Licenciatura em Matemtica

Lema: Se a um nmero composto, ento o menor divisor prprio de n primo.


Demonstrao: Seja d o menor divisor prprio de n . Por definio, d 1 . Se d
fosse composto, ele possuiria um divisor prprio, digamos d0 . Mas d0 |d e d |n
implicam que d0 |n e, como d0 < d , haveria um divisor prprio de n menor que
d , contrariando a sua minimalidade.

A
1

Teorema (Fundamental da Aritmtica): Todo nmero inteiro maior que 1


pode ser escrito como produto de nmeros primos. Em outros termos: dado qual

T
4

quer nmero inteiro a > 1 , existem primos p1 ,..., pk distintos, e inteiros positivos
a1 ,..., ak tais que n = p1a1 ... pkak . Alm disso, a menos da ordem dos nmeros, essa
decomposio feita de maneira nica.
Demonstrao: Se n primo, o resultado vale imediatamente. Seja, ento, n
composto. Pelo lema anterior, o menor divisor prprio de n primo. Vamos cham
-lo de p1 . Podemos, ento, escrever n = p1n1 . Se n1 primo, o resultado vale ime
diatamente. Caso contrrio, seja, ento, n1 composto. Da mesma forma, o menor
divisor prprio de n1 primo. Vamos cham-lo de p2 . Assim, podemos escrever
n = p1n1 = p1 p2n2 . O processo pode ser repetido, e como, a cada passo, ni < ni-1 , ou
seja, forma-se uma sequncia decrescente de inteiros positivos e maiores que 1, ha
ver um momento no qual teremos nm = pm primo e, assim, n = p1 ... pm . Como os
primos obtidos no so necessariamente distintos, podemos contar a quantidade de
vezes que cada primo pi aparece. Vamos chamar essa quantidade de ai e concluir
que n = p1a1 ... pkak .
Quanto unicidade, suponha que n = p1a1 ... pkak e n = q1b1 ...qrbr sejam duas decom
posies em nmeros primos do nmero n . Como p1 primo e divide n = q1b1 ...qrbr ,
pela definio de nmero primo, temos p1 |qi para algum i e, como eles so primos,
devem ser iguais, assim, podemos reordenar os primos da segunda decomposio
para que p1 = q1 . Usando um argumento semelhante, podemos concluir que os ex
poentes devem ser iguais tambm. Da igualdade p1a1 ... pkak = q1b1 ...qrbr , e uma vez que
p1 = q1 e a1 = b1 , podemos concluir que p2a2 ... pkak = q2b2 ...qrbr . Da, basta repetir a
ideia de reordenamento para provar que p2 = q2 e a2 = b2 . O raciocnio repetido
k vezes, quando se esgotam os primos pi o mesmo tendo que acontecer com os qi .
Dessa forma, a quantidade de primos distintos nas duas decomposies a mesma
e, a menos de ordem dos nmeros, elas possuem os mesmos fatores com os mesmos
expoentes, o que conclui a demonstrao.
No sem motivo que o teorema acima recebe o nome de fundamental, pois muitos outros
resultados seguem diretamente dele, alm da simplificao de uma srie de outros problemas.

Teoria dos nmeros

27

EXEMPLO 1A

A decomposio em fatores primos do nmero 72 23.32 . Dessa maneira, se d |72 , ne


nhum primo diferente de 2 e 3 pode dividir d , ou seja, podemos escrever d = 2a 3b , com
a {0,1,2,3} e b {0,1,2} . Assim, temos 4 possibilidades para o valor de a e 3 para b .
Pelo Princpio Fundamental da Contagem, podemos concluir, ento, que 72 possui 4.3 = 12
divisores inteiros positivos.

A
1

EXEMPLO 1B

Como 100 = 22.52 , os divisores positivos de 100 so da forma d = 2a5b , com

T
4

a, b {0,1,2} . Assim, 100 possui 3.3 = 9 divisores positivos.


O nmero 32.54.7 possui 3.5.2 = 30 divisores positivos.
EXEMPLO 1C

Para

determinar

m = 4 .12 .15 ,
3

mero

quantidade
devemos

de

obter

divisores
sua

positivos

fatorao

em

do

primos:

4 .12 .15 = (2 ) .(2 .3) .(3.5) = 2 .2 .3 .3 .5 = 2 .3 .5 . Dessa forma, m possui


3

2 3

10

11.6.4 = 264 divisores positivos.


Como visto nos exemplos, podemos determinar a quanti
dade de divisores positivos de um nmero inteiro de maneira

ATENO!

prtica a partir da sua decomposio em nmeros primos. Ou

Se n < -1 , ento -n um
inteiro maior que 1 e, assim, possui
decomposio em nmeros
primos, de modo que n pode ser
escrito como produto de nmeros
primos vezes 1.

a determinao do mximo divisor comum.

tra utilidade dessa maneira de escrever os inteiros positivos


Se a e b so inteiros positivos e d = (a, b) , claro que
d |a . Assim a decomposio de d possui apenas primos cons
tantes na fatorao de a , no podendo exceder os expoentes
dessa fatorao. O mesmo deve acontecer em relao a b , de

modo que a fatorao de d em primos contm exatamente os primos comuns s fa


toraes de a e de b com o menor expoente que aparecer.
EXEMPLO 2A

Como 108 =22.33 e 120 =23.3.5 , o mximo divisor comum entre 108 e 120 tem
em sua fatorao apenas os primos 2 e 3, que so os fatores comuns a ambos. Alm
disso, se o expoente do 2 fosse maior que 2, o resultado no seria divisor do 108. O
expoente para 3 ser 1, que o mximo possvel para ser divisor de 120. Dessa for
ma, temos (108,120) = 22.3 = 12 .
EXEMPLO 2B

Como 72 = 23.32 e 35 = 7.5 , temos (72,35) = 1 .

28

Licenciatura em Matemtica

Se pusermos os nmeros primos em ordem crescente e escrevermos p1 = 2 ,


p2 = 3 , p3 = 5 e assim sucessivamente, podemos dizer que todo nmero possui

uma decomposio nica da forma n = p1a1 p2a2 ... = pkak , onde ak diferente de 0
se pk |n , e igual a 0, caso contrrio.

k =1

k =1

k =1

k =1

Dessa forma, se a = pkak e b = pkbk , vale dizer que (a, b) = pkgk , onde g k

A
1

o mnimo entre ak e bk .
claro que muito mais se pode extrair da relao de divisibilidade e da decom

T
4

posio em nmeros primos. Continuaremos com esse assunto, dando-lhe mais pro
fundidade, vendo novos conceitos e revisitando outros. Por ora, encerramos essa
primeira aula. At breve.

Teoria dos nmeros

29

Aula 2
Mltiplos

Ol a todos,
Continuamos nosso estudo de Teoria dos Nmeros. Na aula 1, demos nfase aos
divisores de um nmero inteiro. Nesta aula 2, continuamos com o assunto, abordando tambm os seus mltiplos. Veremos alguns critrios de divisibilidade e como
representar nmeros em bases diferentes da base 10.

Objetivos:
Dar prosseguimento ao estudo da divisibilidade.
Relacionar propriedades de divisibilidade e sistema decimal.

31

01

MNIMO
MLTIPLO COMUM

TPICO

OBJETIVOS

Definir e verificar as principais propriedades do


mnimo mltiplo comum entre dois nmeros;
Relacionar mnimo mltiplo comum e mximo
divisor comum.

ara os nmeros inteiros positivos a e b , vimos que a |b quando b = aq


para algum inteiro q , caso em que dizemos que a um divisor de b , e b
um mltiplo de a . O conjunto de mltiplos inteiros positivos do nmero

a denotado por M (a ) e sempre ilimitado superiormente.


Consequentemente podemos verificar, pela transitividade da relao de divisi

bilidade, que a |b se, e somente se, M (b) M (a ) . Dessa relao, obtemos que se
c M (a ) M (b) , ento M (c ) M (a ) M (b) .
Definio 1: Dados os nmeros inteiros positivos a e b , dizemos que o nme
ro m *+ o mnimo mltiplo comum e escrevemos m = [a, b ] quando:
(i) a |m e b|m , ou seja, m M (a ) M (b) ;
(ii) se a |m1 e b|m1 , ento m |m1 , ou seja, m o menor nmero positivo com
a propriedade (i).

32

Licenciatura em Matemtica

EXEMPLO 1A:

Como M (2) = {2,4,6,8,10,...} e M (5) = {5,10,15,20,25,....} , vale [2, 5] = 10.


EXEMPLO 1B:

Observando que 7|721 , podemos dizer que [721, 7] = 721.


Uma vez que para os inteiros positivos a e b sempre verdade que

A
2

ab M (a ) M (b) , o conjunto dos mltiplos comuns a dois inteiros positivos nunca


vazio e, como possui apenas nmeros positivos, possui sempre um elemento mni

T
1

mo, de onde podemos concluir que sempre existe [a, b ] .


Considerando os primos listados em ordem crescente (como no final da aula 1) e

k =1

k =1

as decomposies a = pkak e b = pkbk , vale [a, b ] = pkwk , onde wk o mxi


k =1

mo entre ak e bk . De fato, se m = p , vale a |m e b|m . Alm disso, qualquer


k =1

wk
k

mltiplo simultneo de a e b deve conter todos os fatores primos das suas decom
posies, fazendo com que o valor de m seja o menor possvel.
EXEMPLO 2A:

Como 36 = 2232 e 40 = 235 , vale [36,40] = 23325 = 360 .


EXEMPLO 2B:

Quando dois nmeros so primos entre si, eles no possuem fatores primos em
comum. Assim sendo, seu mnimo mltiplo comum igual ao produto dos dois. Por
exemplo, [16,27] = 16.27 = 432 .
Uma
ros

vez

reais

que

para

vale

mn {a, b }

quaisquer
+

mx {a, b }

nme
=

a+b ,

simples verificar que (a, b).[a, b ] = a.b , para quaisquer inteiros positivos a e b .
Assim, o algoritmo de Euclides fornece, tambm, uma maneira de encontrar o mni
mo mltiplo comum entre dois nmeros, como vemos a seguir.
EXEMPLO 3:

Podemos determinar (60, 36) de acordo com o esquema:


60 = 36 + 24

(60, 36) = (36, 24)

36 = 24 + 12

(36, 24) = (24, 12)

24 = 2.12

(24, 12) = 12

Assim, (60, 36) = 12, mas como (60,36).[60,36] = 60.36 , podemos escrever
[60,36] =

60.36
= 180 .
12

Teoria dos nmeros

33

02
TPICO

OUTRAS BASES

OBJETIVO

Escrever nmeros inteiros em diversas bases.

bserve que, ao escrevermos, por exemplo, o nmero 3272, estamos usando


a base 10, o que significa que podemos represent-lo da seguinte forma:
3272 = 3.103 + 2.102 + 7.101 + 2.10 0 .

Desde cedo aprendemos esse tipo de notao. Uma das consequncias dessa no

tao a propriedade que assegura que, para se multiplicar um nmero inteiro por
10, basta acresentar um 0 direita do nmero. Neste tpico, veremos que possvel
fazer o mesmo para qualquer base.
Teorema: Seja B >1 um nmero inteiro. Todo nmero inteiro positivo a pode
ser expresso, de maneira nica, na forma a = rn B n + ... + r1B + r0 , com 0 rk < B
, para qualquer 0 k < n .
Demonstrao: Comecemos dividindo a por B , obtendo quociente q0 e resto r0 .

34

Licenciatura em Matemtica

Assim, vale dizer que:


a = Bq0 + r0 , com 0 r0 < B .

Dividindo, ento, q0 por B , obtemos quociente q1 e resto r1 . Assim, vale

dizer tambm que:


q0 = Bq1 + r1 , com 0 r1 < B .

Note que q0 > q1 . Repetindo o processo, podemos dividir q1 por B , obtendo

A
2

quociente q2 e resto r2 , onde q1 > q2 .


q1 = Bq2 + r2 , com 0 r2 < B .

Repetimos o processo at que qn = 0 , o que fatalmente acontecer, uma vez

T
2

que a sequncia de quocientes decrescente e formada apenas por nmeros no ne


gativos.

Neste caso, teremos os dois ltimos passos:

qn-2 = Bqn-1 + rn-1 , com 0 rn-1 < B .

qn-1 = Bqn + rn = B.0 + rn = rn , com 0 rn < B .

Por retrossubstituio, podemos fazer:

qn-2 = Bqn-1 + rn-1 = Brn + rn-1

qn-3 = Bqn-2 + rn-1 = B(Brn + rn-1 ) + rn-2 = B 2rn + Brn-1 + rn-2

q -4 = Bqn-3 + rn-3 = B(B 2rn + Brn-1 + rn-2 ) + rn-3 = B 3rn + B 2rn-1 + Brn-2 + rn-3
e nassim
sucessivamente at
q0 = Bq1 + r1 = B(B n-2rn + B n-3rn-1 + ... + Br3 + r2 ) + r1

= B n-1rn + B n-2rn-1 + ... + B 2r3 + Br2 + r1


E, por fim:
a = Bq0 + r0 = B(B n-1rn + B n-2rn-1 + ... + Br2 + r1 ) + r0

= B n rn + B n-1rn-1 + ... + B 2r2 + Br1 + r0


Assim, demonstramos a existncia de tais coeficientes.

Observe agora que r0 o resto da diviso de a por B , sendo, portanto,

unicamente determinado. Alm disso, Br1 + r0 o resto da diviso de a por B 2 ,


fazendo com que r1 seja unicamente determinado. Uma repetio desse argumento
nos levar unicidade da representao.
Por simplicidade, escrevemos a = rn B n + ... + r1B + r0 = (rn ...r1r0 )B e, como de

costume, quando a base omitida, porque estamos usando 10.


EXEMPLO 1A:

Escreva 185 na base 7.


Soluo:

Fazendo divises sucessivas:

Teoria dos nmeros

35

185 = 7.26 + 3

26 = 7.3 + 5.

Assim, temos 187 = 7.(7.3 + 5) + 3 = 3.72 + 5.7 + 3 = (353)7

EXEMPLO 1B:

Escreva 2185 na base 5.

A
2

2185 = 437.5 + 0

437 = 87.5 + 2

87 = 17.5 + 2

T
2

17 = 3.5 + 2

3 = 0.5 + 3

Por substituio, temos:

87 = 17.5 + 2 = (3.5 + 2).5 + 2 = 3.52 + 2.5 + 2

437 = 87.5 + 2 = (3.52 + 2.5 + 2).5 + 2 = 3.53 + 2.52 + 2.5 + 2

2185 = 437.5 + 0 = (3.53 + 2.52 + 2.5 + 2).5 + 0 = 3.54 + 2.53 + 2.52 + 2.5 + 0 = (32220)5
EXEMPLO 1C:

Passe 217 para a base 6

ATENO!
Em outras palavras, o teorema acima
pode ser expresso por:
Para cada par de nmeros a, B ,
com B >1 , existe um nico polinmio
p [x ] , com coeficientes menores
que B e no negativos tais que
a = p( B ) .

217 = 6.36 + 1

36 = 6.6 + 0

6 = 1.6 + 0

1 = 0.6 + 1

Logo 217 = (1001)6 .


Tambm podemos fazer o processo inverso, ou seja, trans
formar um nmero em uma base qualquer para a decimal,
processo este que feito de maneira ainda mais simples.
EXEMPLO 2:

Passe o nmero (432)8 para a base 10.


Soluo:

Vemos, pela definio, que


(432)8 = 4.82 + 3.8 + 2 = 4.64 + 24 + 2 = 292 .

EXEMPLO 3:

Independentemente da base B > 2 , o nmero (121)B um quadrado perfeito, pois


(121)B = 1.B 2 + 2.B + 1 = B 2 + 2.B.1 + 12 = (B + 1)2 , ou seja, o quadrado de um inteiro.

36

Licenciatura em Matemtica

03
TPICO

CONGRUNCIA

OBJETIVOS

Estabalecer a notao de congruncia.


Verificar as
congruncia.

principais

propriedades

de

este tpico, veremos uma maneira interessante de escrever o resto da di


viso entre dois nmeros. A notao que estabeleceremos ser muito til.

Definio 2: Dado o nmero inteiro positivo n , dizemos que os inteiros a e b


so congruentes mdulo n , e representamos por a b(mod n) , quando a - b
um mltiplo de n , ou seja:
a b(mod n) n|a - b
EXEMPLO 1:

Uma vez que 47 3 = 44, que um mltiplo de 4, podemos escrever 44 3(mod 4) .


Podemos verificar tambm que 25 10(mod5) , 49 0(mod7) e 50 2(mod6) .

38

Licenciatura em Matemtica

Proposio 1: Se a b(mod n) , ento a e b deixam o mesmo resto na


diviso por n .
Demonstrao: Escrevendo a = nq1 + r1 e b = nq2 + r2 , com 0 r1 , r2 < n
e usando a definio, temos

a b(mod n) n|a - b $q; a - b = nq .

Assim, por substituio, obtemos

(nq1 + r1 ) - (nq2 + r2 ) = nq , ou seja,

r1 - r2 = nq - nq1 + nq2 = n(q - q1 + q2 ) , de onde conclumos que n|r1 - r2 , e,

A
2

como 0 r1 , r2 < n , obtemos r1 - r2 = 0 , isto , os restos so iguais. Ressalta-se


aqui que a recproca dessa proposio tambm vlida.

T
3

Relembramos que a diviso do inteiro a pelo inteiro positivo n deixa re


sto no negativo e sempre menor que n . Dessa forma, se r tal resto, neces
sariamente 0 r < n . Podemos, ento, resumir essa informao em notao:
"a , $m {0,1,..., n - 1}; a m(mod n) .
Teorema: Dado o inteiro positivo n , a relao de congruncia mdulo n satis
faz as seguintes propriedades:
(i) a a(mod n) , para qualquer inteiro a , ou seja, uma relao reflexiva;
(ii) se a b(mod n) , ento b a(mod n) , para quaisquer inteiros a e b , ou se
ja, uma relao simtrica;
(iii) se a b(mod n) e b c (mod n) , ento a c (mod n) , para quaisquer intei
ros a , b e c , ou seja, uma relao transitiva;

A demonstrao deste teorema imediata, uma vez


que, como provado anteriormente, dois nmeros so

GUARDE BEM ISSO!

congruentes mdulo n quando deixam o mesmo resto


na diviso por n .
Vejamos agora que a relao de congruncia preser
vada por somas e produtos.

Uma relao reflexiva, simtrica


e transitiva chamada de relao
de equivalncia.

Proposio 2: Se a b(mod n) e c d (mod n) , ento a + c b + d (mod n) e


ac bd (mod n) .
Demonstrao:

Por

definio

a b(mod n) n|a - b .

Analogamente

c d (mod n) n|c - d . Sabemos que a soma de dois mltiplos de n tam


bm um mltiplo de n , ou seja, a - b + c - d um mltiplo de n , isto ,

Teoria dos nmeros

39

n|(a + c ) - (b + d ) e, assim, a + c b + d (mod n) .


Se n|a - b , ocorre tambm que n|(a - b)c . Da mesma forma, se n|c - d ,
n|(c - d )b . Usando argumento semelhante, conclumos que, se n|a - b ,
vale dizer tambm que (a - b)c + (c - d )b um mltiplo de n , mas
(a - b)c + (c - d )b = ac - bc + bc - bd = ac - bd , ou seja, n|ac - bd , de onde

A
2

obtemos ac bd (mod n) .
Como concluso da proposio acima, podemos afirmar que a congruncia no

T
3

apenas uma maneira simplificada de dizer que dois nmeros possuem o mesmo resto
na diviso por outro (e assim tambm uma maneira alernativa de dizer que um n
mero mltiplo do outro), mas tambm que uma relao compatvel com as ope
raes de adio e de multiplicao. Obviamente, como a - b = a + (-1)b , conclu
mos que, tambm com a subtrao, podemos operar com nmeros congruentes sem
alterar essa propriedade. Alm disso, como a potncia com expoente natural consi
ste de produto com fatores repetidos, podemos afirmar que se a b(mod n) , ento
a k b k (mod n) . Vejamos agora alguns exemplos de como essa propriedade pode ser
aplicada para simplificar vrios problemas.
EXEMPLO 2A:

Se o resto da diviso do nmero k por 5 2, qual o resto da diviso de 4k + 13


por 5?
Soluo:

Observe que a condio inicial equivalente a k 2(mod5) e, como 4 4(mod5) ,


podemos, pela conservao do resto na multiplicao, afirmar que 4k 8(mod5) .
Por fim, como 13 3(mod5) , somamos para obter 4k + 13 11(mod5) . Assim, o re
sto da diviso de 4k + 13 por 5 o mesmo que o de 11 por 5. A resposta , portanto, 1.
EXEMPLO 2B:

Mostre que no existe inteiro a tal que a 2 2(mod3) .


Soluo:

Em relao congruncia mdulo 3, temos trs possibilidades:


se a 0(mod3) , ento a 2 02 (mod3) , ou seja, a 2 0(mod3) e, claro,
a 2(mod3) ;
se a 1(mod3) , ento a 2 12 (mod3) , ou seja, a 2 1(mod3) e, claro,
a 2(mod3) ;

40

Licenciatura em Matemtica

por fim, se a 2(mod3) , ento a 2 22 (mod3) , ou seja, a 2 1(mod3) e, cla


ro, a 2(mod3) ;
Assim, esgotam-se todas as possibilidade, e podemos afirmar que a equao
a 2(mod3) no possui solues em .
2

EXEMPLO 2C:

A
2

Uma vez que 6 1(mod5) , podemos dizer que 640 1(mod5) . Geralmente,
6 1(mod5) , para qualquer inteiro positivo k .
k

T
3

EXEMPLO 3:
14

Determine o algarismo das unidades de 14 .


Soluo:

Podemos perceber diretamente que o algarismo das unidades de um nmero, na


representao decimal, o resto da diviso deste nmero por 10. Inicialmente, obser
vamos que 14 4(mod10) . Assim, 1414 414 (mod10) , mas 42 = 16 6(mod10) e
43 = 42.4 6.4(mod10) , ou seja, 43 4(mod10) . Geramos, ento, uma sequncia pe
ridica, sendo 4n 4(mod10) se n mpar, e 4n 6(mod10) se n par. Desta feita,
obtemos 414 6(mod10) e, consequentemente, o algarismo das unidades de 1414 6.
Outras informaes a respeito de congruncia sero discutidas nas prximas au
las, nas quais enunciaremos resultados importantes e suas respectivas aplicaes.

Teoria dos nmeros

41

04

CRITRIOS
DE DIVISIBILIDADE

TPICO

OBJETIVO

Analisar critrios segundo os quais um nmero


divide o outro, sem apelar para a diviso direta.

sando a base 10, agora veremos como reconhecer mltiplos de alguns n


meros sem ter que recorrer diviso. Revisitaremos, ento, algumas re
gras, justificando-as.

Divisibilidade por 2: Um nmero divisvel por 2 se, e somente se, quando


escrito na base 10, terminar em um algarismo par, ou seja, em 0, 2, 4, 6 ou 8.
Demonstrao: Dado o nmero a , podemos escrev-lo na base 10 da forma
a = rn ...r1r0 , significando a = rn .10n + ... + r1 .10 + r0 = 10(rn .10n-1 + ... + r1 ) + r0 .
Fazendo k = rn .10n-1 + ... + r1 , temos, ento a = 10k + r0 = 2.5k + r0 . Assim, a
e r0 deixam o mesmo resto na diviso por 2.
EXEMPLO 1:

Os nmeros 23472 e 8008 so divisveis por 2, enquanto 98221 e 507 no so.

42

Licenciatura em Matemtica

Observao: importante ressaltar que os critrios descritos neste tpico esto


sendo enunciados tomando a representao do nmero na base 10; quando a base
for diferente, devem-se fazer os devidos ajustes. O nmero (324)5 , por exemplo,
mpar, enquanto o nmero (11)7 par. (Verifique).
Divisibilidade por 3: Um nmero divisvel por 3 se, e somente se, a soma dos

A
2

seus algarismos na representao em base 10 for divisvel por 3.


Demonstrao: Observe inicialmente que 10 1(mod3) , de onde podemos
concluir que 10 k 1(mod3) para qualquer natural k , de onde tambm obte

T
4

mos r .10 k r (mod3) . Dado o nmero a , podemos escrev-lo na base 10 da for


ma a = rn ...r1r0 , significando a = rn .10n + ... + r1 .10 + r0 . Pelas propriedades da
congruncia, temos a rn + ... + r1 + r0 (mod3) . Assim, a e rn + ... + r1 + r0 dei
xam o mesmo resto na diviso por 3.

EXEMPLO 2:

ATENO!

Para saber se o nmero 276534 divisvel por 3, po


demos somar os algarismos que o compem: 2 + 7 + 6 +
5 + 3 + 4 = 27. Como 27 mltiplo de 3, conclumos que
276543 tambm mltiplo de 3. Realizando teste semel
hante, podemos afirmar que 89332 no mltiplo de 3.
Divisibilidade por 4: Um nmero divisvel por 4
se, e somente se, os dois ltimos algarismos da sua re
presentao na base 10 formarem, na ordem em que

Como 10 1(mod9) , o critrio de


divisibilidade por 3, descrito acima,
tambm vlido para divisibilidade
por 9, ou seja, para verificarmos
se um nmero mltiplo de
9, testamos a soma dos seus
algarismos. Assim, por exemplo,
podemos afirmar que 61758423
mltiplo de 9, porque a soma de
seus algarismos um mltiplo de 9.

aparecerem, um nmero divisvel por 4.


Demonstrao:

Dado

nmero

podemos escrev-lo na base 10 da forma


a = rn .10 + ... + r1 .10 + r0 = 100(rn .10
n

n-2

a,
a = rn ...r1r0 , significando

+ ... + r2 ) + 10r1 + r0 .

Como 100 divisvel por 4, e fazendo k = rn .10n-2 + ... + r2 , temos


a = 100k + r0 = 4.25k + 10r1 + r0 . Assim, a e 10r1 + r0 deixam o mesmo resto
na diviso por 4.
EXEMPLO 3:

Uma vez que 32 mltiplo de 4, podemos dizer que 399287532 mltiplo de 4.


Observao:

Como 1000 = 8.125, podemos proceder de maneira anloga ao visto acima para

Teoria dos nmeros

43

afirmar que um nmero divisvel por 8 quando o nmero formado pelos trs lti
mos algarismos de sua representao decimal for um mltiplo de 8.
Divisibilidade por 5: Um nmero divisvel por 5 se, e somente se, quando
escrito na base 10 terminar em 0 ou 5.
Demonstrao: Dado o nmero a , podemos escrev-lo na base 10 da forma
a = rn ...r1r0 , significando a = rn .10n + ... + r1 .10 + r0 = 10(rn .10n-1 + ... + r1 ) + r0 .

A
2

Fazendo k = rn .10n-1 + ... + r1 , temos, ento, a = 10k + r0 = 5.2k + r0 . Assim, a


e r0 deixam o mesmo resto na diviso por 5.

T
4

EXEMPLO 4:

Os nmeros 9355 e 7530 so mltiplos de 5, enquanto 49873 e 541 no so.


Observao:

Os critrios acima podem ser combinados para se verificar se um nmero mlti


plo de 6, de 10 ou de 15. Por exemplo, como 15 = 3.5, um nmero ser divisvel por
15 se, e somente se, for divisvel por 3 e por 5. Alm disso,
verifica-se facilmente que 660 divisvel por 2, por 3 e por
5, sendo, portanto, um mltiplo de 30.

SAIBA MAIS!
No site http://www.somatematica.
c o m . b r / f u n d a m / c r i t d i v. p h p ,
voc poder rever os critrios de
divisibilidade apontados e conhecer
ainda outros, sobre os quais falaremos
a seguir. Bom estudo!

H critrios semelhantes aos apresentados at aqui que


podem ser aplicados para se testar se um nmero mlti
plo de 7, de 11 ou de outros primos. Em seguida, apresen
taremos um exemplo de como podemos verificar se um n
mero mltiplo de 7, entretanto, como se ver, a diviso
direta pode ser meio mais prtico para essa verificao.

EXEMPLO 5:

Como 10 3(mod7) , podemos, multiplicando por 3 e observando o resto na di


viso por 7, escrever a sequncia:

102 2(mod7) ;

103 6(mod7) ;

10 4 4(mod7) ;

105 5(mod7) , e assim sucessivamente. Usando esses resultados, pode

mos verificar se 3801 divisvel por 7. Faamos 3801 = 3.103 + 8.102 + 1. Assim,
3801 3.6 + 8.2 + 1(mod7) , e como 3.6 + 8.2 + 1 = 35, que um mltiplo de 7,
podemos dizer que 3801 tambm um mltiplo de 7. Pode-se repetir o processo e
verificar que o resto da diviso de 4986 por 7 2.

44

Licenciatura em Matemtica

EXEMPLO 6:

Observando inicialmente que 1001 = 7.11.13, temos 1001 0(mod7) , ou se


ja, 1000 -1(mod7) . Podemos verificar se um nmero divisvel por 7, 11 ou
13 atravs de um teste que ser descrito aqui com o nmero 124397. Uma vez que
124397 = 124.1000 + 397 , temos 124397 124.(-1) + 397(mod7) . Agora, como
397 124 = 273, que um mltiplo de 7, afirmamos que 124397 mltiplo de 7.

A
2

Como 11|273 , podemos dizer, pela mesma ideia, que 124397 no mltiplo de 11.

T
4

Teoria dos nmeros

45

Aula 3

Alguns teoremas sobre Congruncia


Ol aluno(a),

Nesta nossa terceira aula, estudaremos detalhadamente a congruncia de nmeros


inteiros, tratando, por meio de enunciado, demonstrao e consequncias, de trs
importantes resultados, os conhecidos teoremas de Wilson, Fermat e Euler.

Objetivo:
Complementar o estudo de congruncia, dando-lhe mais aprofundamento.

47

01

TEOREMA
DE WILSON

TPICO

OBJETIVOS

Estudar resduos de congruncia;


Enunciar e analisar as consequncias do Teorema
de Wilson.

a ltima aula, estabelecemos que a notao a b(mod n) para os intei


ros a , b e n indica que a e b deixam o mesmo resto na diviso por n
. Por exemplo, verdade que 25 1(mod 4) e 19 4(mod5) , enquanto

a 0(mod n) equivalente a n|a .


Neste primeiro tpico, discutiremos algumas propriedades sobre congruncia e

enunciaremos alguns resultados relevantes.


Comecemos com algumas definies simples.
Definio 1: O conjunto S = {r0 , r1 ,..., rk } um sistema completo de resduos
mdulo n se
(i) ri rj (mod n) i = j , ou seja, dois elementos distintos de S deixam res
tos distintos na diviso por n ;
(ii) para todo inteiro a , tivermos a ri (mod n) para algum ri S .

48

Licenciatura em Matemtica

EXEMPLO 1A:

O conjunto {5, 11, 22, 33, 44} um sistema completo de resduos mdulo 5.
EXEMPLO1B:

Para qualquer inteiro n >1 , o conjunto {0,1,..., n - 1} um sistema completo de


resduos mdulo n .

A
3

Algo que podemos verificar que qualquer sistema completo de resduos m


dulo n possui exatamente n elementos. De fato, se S = {r0 , r1 ,..., rk } um sistema

T
1

completo de resduos mdulo n , cada um dos nmeros do conjunto {0,1,..., n - 1}


congruente a um dos ri S . Logo, k n . Reciprocamente, cada elemento de S
congruente a um nmero de {0,1,..., n - 1} , que , tambm, um sistema completo de
resduos mdulo n , de onde obtemos que n k , o que resulta em k = n .
Definio 2: Dado o nmero inteiro a , dizemos que a-1 um inverso de
a mdulo n se a.a-1 1(mod n) .
EXEMPLO 2A:

Como
to

7.3
na

21

diviso

por

e
5,

21
vale

deixa

res

7.3 1(mod5) ,

e podemos dizer que 3 um inverso de 7 mdulo 5.


EXEMPLO 2A:

Como 40 mltiplo de 8, vale 40.b 0(mod8) , para


qualquer inteiro b , de onde conclumos que 40 no pos
sui inverso mdulo 8. Mais geralmente, nenhum mltiplo
de n possui inverso mdulo n .
Podemos investigar a existncia de inversos mdulo n

ATENO!
Na definio acima, usamos o artigo
indefinido, pois, caso exista, o inverso
mdulo n de um nmero no
nico, por exemplo, 7.3 1(mod5)
e 7.8 1(mod5) , de onde podemos
dizer que 8 tambm um inverso de 7
mdulo 5. Entretanto, como veremos
adiante, os inversos mdulo n de
um mesmo nmero so congruentes
mdulo n .

de acordo com o que segue.


Proposio 1: O nmero a possui inverso mdulo n se, e somente se,
(a, n) = 1 .
Demonstrao: O inverso mdulo n uma soluo para a equao
ax 1(mod n) . Supondo que exista tal soluo, deve haver um inteiro y tal que
ax - 1 = ny , ou seja, ax - ny = 1 , que uma equao diofantina linear nas vari
veis x e y , que, de acordo com o exposto na aula 1 (tpico 3), possui soluo
se, e somente se, (a, n)|1 , o que vale apenas quando (a, n) = 1 .

Teoria dos nmeros

49

EXEMPLO 3A:

Os nmeros 2, 4, 5, 6, 8 e 10 no possuem inverso mdulo 10.


EXEMPLO 3B:

Os nmeros 1, 3, 7 e 9 so inversos mdulo 10 de 1, 7, 3 e 9, respectivamente.


EXEMPLO 3C:

A
3

Para o nmero primo p , todos os nmeros 1, 2, 3, p -1 possuem inverso m


dulo p .

T
1

Proposio 2: Se b e c so inversos de a mdulo n , ento b c (mod n) .


Demonstrao: Pela definio, a hiptese diz que ab 1(mod n) e ac 1(mod n) .
Subtraindo essas duas congruncias, obtemos a(b - c ) 0(mod n) , ou seja,
n|a(b - c ) , mas, como (a, n) = 1 condio necessria para existir o inverso de a
mdulo n , obtemos n|b - c , que conclui a demonstrao.
Assim, quando um nmero a possuir inverso mdulo n , a quantidade de in
versos infinita, contudo, dentro de um sistema completo de resduos mdulo
n , esse inverso nico.
Especificamente, se p um nmero primo, ento a ser seu prprio inverso m
dulo p se, e somente se, a 1(mod p) ou a -1(mod p) , pois, se a 2 1(mod p) ,
ento vale p|a 2 - 1 = (a + 1).(a - 1) , ou seja, p|a + 1 ou p|a - 1 , casos que impli
cam a -1(mod p) e a 1(mod p) , respectivamente. A recproca imediata.
EXEMPLO 4:

Sabemos que o conjunto S ={0,1,...,10} um sistema


completo de resduos mdulo 11. O nmero 0, obviamen

SAIBA MAIS!

te, no possui inverso mdulo 11. Como 11 primo, todos

No site http://www.dec.ufcg.edu.
br/biografias/JohnWiso.html,
voc
pode obter mais informaes sobre o
matemtico John Wilson. Confira!

os outros possuem inverso mdulo 11, nico no conjunto,


e apenas 1 e 10 so seus prprios inversos. Assim, 2 e 6 so
inversos mdulo 11, o mesmo acontecendo com os pares 3
e 4; 5 e 9; e 7 e 8.

Podemos, agora, enunciar e ter uma demonstrao simples de um teorema que


leva o nome do matemtico ingls John Wilson (1741 1793).

50

Licenciatura em Matemtica

Teorema de Wilson 1: Se p primo, ento ( p - 1)! -1(mod p) .


Demonstrao: O caso p = 2 de imediata verificao. Entre os nmeros
1,2,..., p - 1 , apenas 1 e p -1 so seus prprios inversos mdulo p . Os demais,
2,..., p - 2 , podem ser agrupados em pares cujo produto congruente a 1 m

dulo p . Isso se deve ao fato de que eles possuem inverso mdulo p , diferen

A
3

te de si mesmo e pertencente ao conjunto, ou seja, se a {2,..., p - 2} , existe


b {2,..., p - 2} , com b a , tal que ab 1(mod p) . Se multiplicarmos todas es
sas congruncias sem repetir os nmeros, obteremos 2.3.....( p - 2) 1(mod p) .

T
1

Se multiplicarmos esta ltima congruncia pela imediata p - 1 -1(mod p) , ob


teremos 2.3.....( p - 2).( p - 1) -1(mod p) , isto , ( p - 1)! -1(mod p) .

Teorema de Wilson 2: Se (n - 1)! -1(mod n) , ento n primo.


Demonstrao: Vamos supor que (n - 1)! -1(mod n) . Se a < n , aparece
a no clculo de (n - 1)! , de onde conclumos que (n - 1)! 0(mod a ) . Se ti
vermos a |n , da hiptese n|(n - 1)!+ 1 e da transitividade da divisibilidade,
a |(n - 1)!+ 1 , ou seja, (n - 1)!+ 1 0(mod a ) . Subtraindo (n - 1)!+ 1 0(mod a )
e (n - 1)! 0(mod a ) , obtemos 1 0(mod a ) , mas isso somente possvel se
a = 1 . Assim, 1 o nico inteiro positivo menor que n que divisor de n , de
onde conclumos que n primo.
Poderamos ter enunciado que (n - 1)! -1(mod n) se, e somente se, n pri
mo. A fragmentao foi feita apenas por carter didtico. Com isso, obtemos um
no muito objetivo teste de primalidade.
EXEMPLO 5A:

Mostre que, se n um mltiplo de 5, o resto da diviso de (n + 1)(n + 2)(n + 3)(n + 4)


por 5 4.
Soluo:

Como

n 0(mod5) ,

n + 3 3(mod5)
ncias,

obtemos

ento

n + 4 4(mod5) .

n + 1 1(mod5) ,
Multiplicando

n + 2 2(mod5) ,
as

quatro

(n + 1)(n + 2)(n + 3)(n + 4) 1.2.3.4(mod5) ,

congru
isto

(n + 1)(n + 2)(n + 3)(n + 4) 4!(mod5) , mas, como 5 primo, temos, pelo teore
ma de Wilson, (5 - 1)! -1(mod5) . Como o resto deve ser um nmero positivo e
-1 4(mod5) , obtemos que o resto da diviso de (n + 1)(n + 2)(n + 3)(n + 4) por 5 4.

Teoria dos nmeros

51

EXEMPLO 5B:

O produto de uma sequncia de 16 nmeros inteiros consecutivos pode ou


no possuir um mltiplo de 17. Caso possua, o produto de todos eles deixar res
to 0 na diviso por 17. Do contrrio, teremos um sistema completo de resduos
mdulo 17, e, se multiplicarmos todos eles, obteremos um nmero que, quando
dividido por 17, deixa o mesmo resto que 16! = (17 1)!, mas, pelo teorema de

A
3

teiros consecutivos deixa resto 0 ou 16 na diviso por 17, de modo que a equao

T
1

EXEMPLO 6:

Wilson, esse resto congruente a -1. Assim, o produto de dezesseis nmeros in


n(n + 1)(n + 2)...(n + 15) 10(mod17) no possui soluo.

Como resultado embutido na demonstrao do teorema de Wilson, podemos afir


mar que, se p primo, ento ( p - 2)! 1(mod p) , de modo que (11 - 2)! = 9! deixa
resto 1 na diviso por 11. Nesses termos, conclumos que 9!- 1 um mltiplo de 11.
Analogamente 23|21!- 1.

52

Licenciatura em Matemtica

02

TEOREMA
DE FERMAT

TPICO

OBJETIVOS

Complementar o estudo sobre resduos;


Enunciar e estudar as consequncias do Pequeno
Teorema de Fermat.

amos aqui nos aproveitar dos resultados obtidos no tpico anterior para
provar um resultado que leva o nome do matemtico francs Pierre de Fer
mat (1601 1665). Um resultado forte que Fermat conjecturou e somente

foi provado na dcada de 1990 pelo matemtico ingls Andrew Wiles (1953 ), diz
que, se n > 2 um inteiro, a equao x n + y n = z n no possui soluo no trivial. A
demonstrao desse fato envolveu esforos de muitos pesquisadores, construes so
fisticadas e associaes de muitos ramos da Matemtica. Por comparao, o resultado
que segue enunciado, em muitos textos, como o Pequeno Teorema de Fermat, mas
ele tem consequncias muito interessantes na Teoria dos Nmeros.
Teorema de Fermat: Se p um primo que no divide a , ento
a

54

p-1

1(mod p) .

Licenciatura em Matemtica

Demonstrao: Para comear, S = {0, a,2a,3a,...,( p - 1)a} um sistema


completo de resduos mdulo p . De fato, ab ac (mod p) e (a, p) = 1 condu
zem a b c (mod p) , mas b, c {0,1,2,...,( p - 1)} , que um sistema completo de
resduos mdulo p , logo b = c . Assim, os nmeros a,2a,3a,...,( p - 1)a deixam
restos 1,2,...,( p - 1) na diviso por p , no necessariamente nessa ordem. Temos,
ento, a congruncia:

a.2a.3a.....( p - 1)a 1.2.....( p - 1)(mod p) , que pode ser simplificada por

a p-1 .( p - 1)! ( p - 1)!(mod p) .

A
3
T
2

Mas p no divide ( p - 1)! , de onde conclumos que ( p,( p - 1)!) = 1 e os fatores


( p - 1)! podem ser cancelados da ltima congruncia, de onde obtemos a concluso:
a p-1 1(mod p) .

Com a hiptese de que p no um divisor de a , temos, ento, a p-1 1(mod p) .


Se multiplicarmos a congruncia pela imediata
a a(mod p) . Agora, se p|a , ento p|a.(a
p

p-1

a a(mod p) , obteremos

- 1) , ou seja, p|a p - a , e obtemos

o mesmo resultado a p a(mod p) . Assim, podemos enunciar o seguinte resultado,


sem hipteses de divisibilidade:
Proposio: Se p primo e a um inteiro positivo qualquer, ento
a a(mod p) .
p

EXEMPLO 1A:

Encontre o resto da diviso de 2120 por 13.


Soluo:

J que 13 primo e no um divisor de 2, podemos usar o Teorema de Fer


mat para concluir que 213-1 1(mod13) , ou seja, 212 1(mod13) . Da obtemos

(212 )

10

110 (mod13) , isto , 2120 1(mod13) e, assim, o resto procurado 1.

EXEMPLO 1B:

Encontre o resto da diviso de 5110 por 19.


Soluo:

Observe que no podemos usar exatamente o mesmo raciocnio do exemplo 1a, pois 18
no um divisor de 110, mas podemos escrever 110 = 18.6 + 2 e, assim, 5110 = (518 ) .52 .
6

J que 19 primo e no um divisor de 5, podemos usar o Teorema de Fermat para con


cluir que 519-1 1(mod19) , ou seja, 518 1(mod19) . Da obtemos (518 ) 16 (mod19) ,
6

isto , 5108 1(mod19) . Alie-se a isso a congruncia facilmente verificvel 52 6(mod19)


e conclumos 5110 6(mod19) . O resto procurado , portanto, 6.

Teoria dos nmeros

55

03
TPICO

TEOREMA
DE EULER

OBJETIVOS

Definir a funo totiente de Euler e estudar suas


caractersticas;
Generalizar o teorema de Fermat para todos os
inteiros.

objeto principal deste tpico um resultado de


vido ao matemtico suo Leonhard Euler (1707

SAIBA MAIS!

1783) e trata de uma generalizao do teorema

No link www.mat.uc.pt/~jfqueiro/
TN5.pdf, voc pode encontrar mais
informoes sobre o teorema de Euler

de Fermat apresentado anteriormente. Antes de enunci


-lo, definiremos uma funo especial. A primeira de uma
srie de funes as funes aritmticas que sero estu
dadas em uma aula posterior

Definio 2: A funo f , chamada de totiente de Euler, associa a cada nme


ro inteiro positivo n a quantidade de inteiros positivos relativamente primos com
n . Mais precisamente, f :{1,2,...} {1,2,...}, definida por f(n) , a quantidade
de elementos do conjunto {m ;0 < m n e (m, n) = 1} .

56

Licenciatura em Matemtica

EXEMPLO 1A:

Como os nmeros inteiros positivos que so menores que 12 e relativamente pri


mos com 12 so 1, 5, 7 e 11, vale f(12) = 4 . Analogamente, os nmeros inteiros po
sitivos que so relativamente primos com 15 so 1, 2, 4, 6, 7, 8, 11, 13 e 14, de modo
que f(15) = 9 .
EXEMPLO 1B:

A
3

Para qualquer primo p , todos os inteiros positivos menores que p so relativa


mente primos com p . Assim, vale f( p) = p - 1 .

T
3

EXEMPLO 1C:

A equao f(n) = n possui apenas a soluo n = 1 , pois, do contrrio, f(n) < n .


EXEMPLO 1D:

Podemos determinar f(100) por meio de um crivo semelhante ao de Erasttenes.


Para tanto, da lista dos nmeros menores que 100, riscamos todos aqueles que sejam
mltiplos dos mesmos divisores primos de 100, a saber: 2 e 5. Se listarmos os intei
ros positivos de 1 a 100 e eliminarmos os mltiplos de 2, sobram apenas 50 nmeros.
Nesse primeiro passo, j foram eliminados os mltiplos de
5 que terminam em 0, ficando para serem eliminados, no
segundo passo, apenas os que terminam em 5. So eles: 5,

ATENO!

15, 25, ..., 95, num total de 10. Assim, riscando esses 10
nmeros dos 50 que haviam restado ao final do primeiro
passo, sobram 40, que o valor de f(100) .
Um raciocnio semelhante ao empregado no exemplo 1d
pode ser usado para determinar o valor de f(n) , entretanto
esse procedimento pode ser bem trabalhoso. Quando estu
darmos as funes aritmticas, aprenderemos um mtodo
mais rpido para essa determinao. Por ora, ficamos com o

Vimos, no tpico 1, que um


nmero possui inverso mdulo n
se, e somente se, ele e n forem
relativamente primos. Podemos
usar essa propriedade para definir,
de forma equivalente, que f(n)
a quantidade de inteiros positivos
menores que n que possuem
inverso mdulo n .

que pode ser obtido diretamente da definio.


Definio 3: Dado o inteiro positivo n , o conjunto S = {r1 ,..., rf ( n ) } um
sistema reduzido de resduos mdulo n se os elementos forem dois a dois incon
gruentes mdulo n e forem todos relativamente primos a com n . Mais formal
mente, S = {r1 ,..., rf ( n ) } um sistema reduzido de resduos mdulo n quando:
(i) (ri , n) = 1, "i ;
(ii) ri rj (mod n) i = j .

Teoria dos nmeros

57

Assim, uma maneira de obter um sistema reduzido de resduos mdulo n retirar


de um sistema completo de resduos todos aqueles que no forem relativamente primos
com n . De modo a abranger todas as possibilidades, da maneira como foi definido, um
sistema reduzido de resduos mdulo n deve ter f(n) elementos.
EXEMPLO 2A:

O conjunto S ={1,5,7,11} um sistema reduzido de resduos mdulo 12.

A
3

EXEMPLO 2B:

Para qualquer primo p , o conjunto {1,2,..., p - 1} um sistema reduzido de re

T
3

sduos mdulo p .
Agora vamos ao objetivo principal do tpico.
Teorema de Euler: Para os inteiros relativamente primos a e n , vale
a

f(n)

1(mod n) .

Demonstrao: Essencialmente, a prova a mesma que a feita do Teorema de


Fermat. Aqui apenas ajustaremos para um sistema reduzido de resduos. Comece
mos verificando que, se S = {r1 ,..., rf ( n ) } um sistema reduzido de resduos m
dulo n , o conjunto S ' = {a.r1 ,..., a.rf ( n ) } tambm o . Para tal, basta observar que,
sendo todos os ri relativamente primos com n e o mesmo acontecendo com a ,
ento (a.ri , n) = 1 . Alm disso, se a.ri a.rj (mod n) , mais uma vez apelando para
(a, n) = 1 , obtemos ri rj (mod n) . Entretanto, dentro de um sistema de resduos,
isso somente ocorre quando ri = rj . Observado isso, fica claro que o produto dos
elementos de S deixa o mesmo resto da diviso por n que o produto dos elemen
tos de S ' , ou seja, a.r1 .a.r2 .....a.rf ( n ) r1 .....rf ( n ) (mod n) , que o mesmo que:

a f ( n ) .r1 .r2 .....rf ( n ) r1 .....rf ( n ) (mod n)

E como (n, r1 .....rf ( n ) ) = 1 , o fator r1 .....rf ( n ) pode ser cancelado na congru


ncia para obtermos a f ( n ) 1(mod n) , que o resultado desejado.
Bastar fazer n primo e observar o exemplo 1b para obter o Teorema de Fermat
como um corolrio do Teorema de Euler.
EXEMPLO 3A:

Encontre o algarismo das unidades de 7100 , quando representado na base 10.


Soluo:

O algarismo das unidades, no sistema decimal, nada mais que o resto da diviso
do nmero por 10. Observe que f(10) = 4 e (7,10) = 1 , da, pelo Teorema de Euler,
74 1(mod10) , de onde conclumos que 7100 = (74 ) 125 (mod10) . Assim, a repre
25

58

Licenciatura em Matemtica

sentao decimal de 7100 termina em 1.


EXEMPLO 3B

Podemos, tambm usando o Teorema de Euler, determinar o algarismo das deze


nas de 2142 , j que os dois algarismos mais direita na representao decimal de um
nmero formam o resto da diviso do nmero por 100. Vimos que f(100) = 40 e,

A
3

alm disso, (100,21) = 1 . Logo 2140 1(mod100) . Multiplicando essa congruncia


por 212 , obtemos 2142 212 (mod100) . Mas 212 = 441 . Assim, 2142 41(mod100)
e o algarismo das dezenas 4.

T
3

Teoria dos nmeros

59

Aula 4

Funes aritmticas - parte I


Caro (a) aluno (a),

Nesta nossa quarta aula, daremos continuidade ao estudo de funes especiais que
expressam alguma propriedade aritmtica do nmero e cujo domnio o conjunto
dos inteiros positivos. De destaque, temos a funo f de Euler, a qual j comeamos
a estudar na aula 3, a funo m de Mbius, e as funes t e s .
No decorrer desta aula, apresentaremos as funes e algumas de suas principais
propriedades, ilustraremos com exemplos e estabeleceremos relaes entre elas. Na
aula 5, daremos continuidade a esse estudo.

Objetivos:
Conhecer as funes aritmticas mais importantes;
Obter meios diretos de determinao de imagem por essas funes.

61

01
TPICO

AS FUNES t E

OBJETIVOS

Reconhecer as funes aritmticas t e s ;


Definir funo multiplicativa.

omearemos o nosso estudo de funes aritmticas

ATENO!

apresentando a funo que associa a cada nmero

De acordo com o que estudamos


no comeo da primeira aula, os
divisores inteiros de um nmeros
sempre vm aos pares. Desse modo,
basta estudar os divisores positivos,
pois, se 3 divisor de um nmero,
ganhamos automaticamente o 3.
Uma formulao mais precisa da
definio ao lado seria t (n) =

inteiro positivo a quantidade dos seus divisores

inteiros positivos. Uma maneira de contar elementos de um


conjunto somar 1 para cada vez que esse elemento apare
cer. Assim, podemos enunciar:

1 .
d|n
d >0

Por simplicidade (para evitar excesso


de notao), consideraremos a partir
daqui apenas os divisores positivos.

62

Licenciatura em Matemtica

Definio 1: Denotaremos pela letra grega t (tau)


a funo que, para cada inteiro positivo n , associa o
valor t (n) = 1 . Equivalentemente, podemos colocar
d|n

t (n) = # D(n) , onde # representa a quantidade de ele


mentos do conjunto.

EXEMPLO 1A:

Como os divisores de 20 so 1, 2, 4, 5, 10 e 20, vale que t (20) = 6 .


EXEMPLO 1B:

Decorre diretamente da definio de nmero primo que t ( p) = 2 , para qualquer primo p .


EXEMPLO 1C:

A
4

A tabela abaixo lista os valores de t(n) para os 12 primeiros inteiros positivos.


n

10

11

12

t(n)

T
1

Do Teorema Fundamental da Aritmtica, todo nmero inteiro positivo maior que


1 pode ser escrito de forma nica como produto de pri
mos. Seja, ento, n = p1a1 . p2a2 ..... pkak . Pela transitividade
da divisibilidade, um divisor de n tem fatorao em pri

GUARDE BEM ISSO!

mos que aparecem na fatorao de n , de modo que pode


mos listar todos os divisores de um nmero pela escolha
dos expoentes. Para cada pi , o expoente pode variar de 0
at ai , sendo, ao todo, ai +1 possibilidades. Pelo Prin
cpio Fundamental da Contagem (conforme estudado em
Matemtica Bsica II), a quantidade de divisores de n se

Como os divisores de um nmero


nunca so maiores que o prprio
nmero, a quantidade de divisores
tambm no passa desse nmero.
Assim, vale t (n) n e a igualdade
somente ocorre para os nmeros 1 e 2.

r, ento, t (n) = (a1 + 1)(a2 + 1)...(ak + 1) .


EXEMPLO 2A:

Se p primo, vale t ( p k ) = k + 1 , para qualquer inteiro positivo k .


EXEMPLO 2B:

Determine o menor nmero inteiro positivo n para o qual se tenha t (n) = 5 .


Soluo:

Se

n = p1a1 . p2a2 ..... pkak ,

equao

t (n ) = 5

equivalente

(a1 + 1)(a2 + 1)...(ak + 1) = 5 . Mas como 5 primo, o produto do primeiro membro


tem um fator igual a 5 e os outros iguais a 1, de modo que, assim, a1 = 4 e todos os
outros expoentes so nulos, ou seja, n = p14 para algum primo p. De modo a minimi
zar o valor de n , consideramos o menor primo. Dessa forma, o menor nmero inteiro
positivo n para o qual se tenha t (n) = 5 n = 24 = 16 .
Outra funo aritmtica interessante que est relacionada com os divisores de
um nmero aquela que associa a cada inteiro positivo a soma dos seus divisores.

Teoria dos nmeros

63

Definio 2: Denotaremos pela letra grega s (sigma) a funo que, para ca


da inteiro positivo n , associa o valor s (n) = d .
d|n

EXEMPLO 3A:

Como os divisores positivos de 10 so 1, 2, 5 e 10, verdadeiro que s (10) = 18 .

A
4

EXEMPLO 3B:

Se p um nmero primo, seus nicos divisores positivos so 1 e p . Dessa forma,


para qualquer nmero primo p vale s ( p) = p + 1 .

T
1

EXEMPLO 3C:

A tabela abaixo lista os valores de s(n) para os 12 primeiros inteiros positivos.


n

10

11

12

s(n)

12

15

13

18

12

28

Se um nmero da forma p k , com p primo e k in


teiro positivo, seus nicos divisores so 1, p, p2 ,..., p k . As
sim, s ( p k ) = 1 + p + p2 + ... + p k . Temos aqui a soma dos

ATENO!

k +1 primeiros termos de uma progresso geomtrica de

Se d |n , vale que d n . Dessa forma,


podemos concluir que s (n) t (n).n .
Alm disso, s (n) n e as igualdades
somente valem para n = 1

razo p e primeiro termo igual a 1. Considerando que a so


ma dos n primeiros termos de uma progresso geomtrica
com primeiro termo a1 e razo q 1 vale Sn =
podemos concluir que s ( p k ) =

p k +1 - 1
.
p -1

a1 (qn - 1)
,
q -1

EXEMPLO 4:

Uma vez que 512 = 29 e com base no exposto acima, podemos calcular
210 - 1
s (512) = s (29 ) =
= 1023 .
2 -1
Antes de tratarmos de outras propriedades interessantes das funes s e t , ve
jamos o caso geral das funes multiplicativas.
Definio 3: A funo f , cujo domnio o conjunto dos inteiros positivos,
dita multiplicativa quando f (m.n) = f (m). f (n) sempre que (m, n) = 1 , ou se
ja, m e n so relativamente primos. Quando a propriedade f (m.n) = f (m). f (n)
for vlida sempre, a funo dita completamente multiplicativa.

64

Licenciatura em Matemtica

EXEMPLO 5A:

Como 1 o elemento neutro para o produto, a funo (constante) f (n) = 1 para


qualquer n completamente multiplicativa.
EXEMPLO 5B:

A funo identidade f (n) = n completamente multiplicativa.

A
4

Proposio 1: A funo t multiplicativa.


Demonstrao: Considere os nmeros inteiros positivos m e n , com fatoraes

T
1

em primos m = p1a1 . p2a2 ..... pkak e n = q1b1 .q2b2 .....qrbr . Nessas condies, temos

t (m) = (a1 + 1)(a2 + 1)...(ak + 1) e t (n) = (b1 + 1)(b2 + 1)...(br + 1) . Se m e n


forem relativamente primos, os primos de uma e de outra decomposio so distintos,
a
de modo que m.n = p1a1 . p2a2 ..... pk k .q1b1 .q2b2 .....qrbr e no h simplificaes nessa

fatorao. Assim, temos:


t (mn) = (a1 + 1)(a2 + 1)...(ak + 1)(b1 + 1)(b2 + 1)...(br + 1) =


= t (m)t (n).

Uma maneira alternativa de obter esse mesmo resultado pela verificao de que,
quando nmeros so relativamente primos, os divisores do produto podem ser obti
dos pela multiplicao dos divisores dos dois nmeros, e, novamente pelo Princpio
Fundamental da Contagem, a quantidade de divisores de mn ser t (m)t (n) .
Observao: Como t (1) = 1 , o caso em que um dos nmeros envolvi
dos no produto 1, no analisado na demonstrao acima, se torna bvio, pois
t (1.m) = t (m) = 1.t (m) = t (1)t (m) . Dessa forma, omitiremos este caso tambm nas
demonstraes sobre a multiplicabilidade das demais funes estudadas nesta aula.

Proposio 2: A funo s multiplicativa.
Demonstrao: Consideremos m e n nmeros inteiros relativamente pri
mos. Pela definio, temos s (mn) = d . Entretanto, uma vez que os nmeros
d|mn

so relativamente primos, cada divisor de mn pode ser escrito como produto de


um divisor de m por um divisor de n . Assim, podemos escrever:

s (mn) = d = d1d2 = d1d2 = d1 . d2 = s (m)s (n) .


d|mn

d1|m
d2|n

d1|m d2|n

d1|m

d2|n

Assim, obtemos que s multiplicativa.

Teoria dos nmeros

65

EXEMPLO 6:

Como (9,10) = 1 , valem


t (90) = t (9.10) = t (9)t (10) = 3.4 = 12

e s (90) = s (9.10) = s (9)s (10) = 13.18 = 234 .


Observao: As funes t e s so multiplicativas, mas no so completamen

A
4

te multiplicativas. Veja, por exemplo, que t (12) = 6 , enquanto t (2).t (6) = 2.4 = 8 .
Como consequncia da multiplicatividade da funo s , se n = p1a1 . p2a2 ..... pkak a
decomposio em primos do inteiro positivo n , temos

T
1

s (n )

= s ( p1a1 . p2a2 ..... pkak ) =


= s ( p1a1 ).s ( p2a2 ).....s ( pkak ).

p k +1 - 1
obtida antes do exemplo 4, conse
p -1
guimos uma frmula para calcular o valor de s(n) . Temos, ento:
Em seguida, pela expresso s ( p k ) =

s (n ) =

p1a1 +1 - 1 p2a2 +1 - 1
pak +1 - 1
.
..... 1
.
p1 - 1
p2 - 1
pk - 1

EXEMPLO 7:

Determine o valor de s(72) .


Soluo:

Pelo uso da fatorao 72 = 23.32 . De acordo com a frmula anterior, obtemos:


s (72)

= s (23.32 ) = s (23 ).s (32 ) =

23+1 - 1 32+1 - 1 24 - 1 33 - 1
.
=
.
=
2 -1
3 -1
1
2
27 - 1
= 16 - 1.
= 15.13 = 195.
2
Outra maneira, talvez mais trabalhosa, seria listar todos os divisores de 72 e fazer

sua soma, ou seja, 1 + 2 + 3 + 4 + 6 + 8 + 9 + 12 + 18 + 24 + 36 + 72 = 195 .


Com esse estudo das funes t e s , encerramos o tpico. No prximo, voltare
mos a estudar a funo f definida na aula 3.

66

Licenciatura em Matemtica

02
TPICO

A FUNO

DE EULER

OBJETIVO

Aprofundar o estudo da funo totiente.

a aula 3, apresentamos a funo que associa a cada nmero inteiro po


sitivo a quantidade de inteiros positivos menores ou iguais a ele com os
quais ele relativamente primo. Recapitulando esta definio:

Definio: Denotaremos pela letra grega f (phi) a funo que, para cada
inteiro positivo n , associa o valor f(n) = #{k ;0 < k < n e (k, n) = 1} .

EXEMPLO 1A:

A tabela abaixo lista os valores de f(n) para os 12 primeiros inteiros positivos.


n

68

Licenciatura em Matemtica

10

11

12

f(n)

10

EXEMPLO 1B:

Para qualquer nmero primo p , vale f( p) = p - 1 .


Pelo que vimos no tpico anterior, se uma funo aritmtica multiplicativa,

A
4

basta que saibamos como ela age nos nmeros primos e nas potncias de primos. An
tes de verificar que f uma funo multiplicativa, vejamos como encontrar o valor
de f(n) , no caso de n ser potncia de um primo, digamos n = p k . Como o nico

T
2

divisor primo de p k p , para que (m, n) = 1 , no deve aparecer o primo p na fa


torao de m , ou seja, no pode ser um mltiplo de p , mas os mltiplos de p que
so menores ou iguais a p k so p,2 p,3 p,..., p k = p k-1 p , no total, portanto, de p k-1 .
Assim, conclumos que f( p k ) = p k - p k-1 .
Proposio 3: A funo f multiplicativa.
Demonstrao: Considere os inteiros positivos m e n , relativamente primos.
Por definio, f(mn) a quantidade de nmeros menores que mn e que lhe so
relativamente primos. Comecemos por separar os elementos de A ={1,2,..., mn}
em suas classes de congruncia mdulo m , isto , agruparemos em

A1 = {a ;0 < a mn e a 1(mod m)} ;

A2 = {a ;0 < a mn e a 2(mod m)} ;

...

Am = {a ;0 < a mn e a m(mod m)} .

Uma vez que {1,2,..., m} um sistema completo de resduos mdulo m , to


dos os conjuntos descritos acima so disjuntos e a unio de todos eles o pr
prio A ={1,2,..., mn} . Para cada r = 1,2,...m , veja que, se m e r no forem
relativamente primos, nenhum dos elementos de Ar ser relativamente primo
com mn , pois se m e r possurem um divisor comum diferente de 1, ento os
nmeros da forma km + r , que so os elementos de Ar , sero divisveis por es
se divisor comum, logo sero divisveis por m e, pela transitividade da divisi
bilidade, sero tambm divisveis por mn . Assim, Ar s conter algum nme
ro relativamente primo com mn se (m, r ) = 1 . Mas de 1 a m , sabemos que h
f(m) destes nmeros, pela definio da funo totiente.
Estudemos agora dentro de cada um dos f(m) conjuntos Ar destacados na
primeira etapa quantos elementos so relativamente primos com mn . Observe
que os elementos de cada Ar so da forma km + r . Como m e n so relativa
mente primos, cada conjunto Ar um sistema completo de resduos mdulo n ,

Teoria dos nmeros

69

havendo, portanto, f(n) elementos que so relativamente primos com n e, as


sim, com mn .
Dessa forma, temos f(m) conjuntos e em cada um deles h f(n) elementos
relativamente primos com mn . Assim, no total, temos f(m)f(n) elementos do
conjunto A ={1,2,..., mn} , que so relativamente primos com mn . Mas esta

A
4

exatamente a definio de f(mn) e por ela conclumos que f(mn) = f(m)f(n) .


EXEMPLO 2:

Como 81 = 34 , podemos escrever f(81) = f(34 ) = 34 - 33 = 81 - 27 = 54 .

T
2

EXEMPLO 3:

Como (9,10) = 1 , vale f(90) = f(9.10) = f(9).f(10) = 6.4 = 24 .


Agora que sabemos que a funo f multiplicativa e sabemos calcular os seus
valores para qualquer potncia de primo, podemos usar o Teorema Fundamental da
Aritmtica para estabelecer uma frmula geral para f(n) .
Observe, inicialmente, que, para

primo, obtivemos anteriormente


1
f( p k ) = p k - p k-1 , que pode ser reescrito como f( p k ) = p k 1 - . Considere, en
p

ak
a1
a2
to, a fatorao em primos n = p1 . p2 ..... pk . Usando a multiplicabilidade da fun
p

o f , podemos fazer:
f(n )

= f( p1a1 . p2a2 ..... pkak ) =


= f( p1a1 )f( p2a2 )...f( pkak ) =

1
1
1
= p1a1 1 - p2a2 1 - ... pkak 1 - =
p1
p2
pk

1
1
1
= p1a1 p2a2 ... pkak .1 - 1 - ...1 - =

p1
p2
pk

1
1
1
= n.1 - 1 - ...1 - .

p1
p2
pk

Assim, para encontrarmos o valor de f(n) , basta que multipliquemos n pela ex

1
presso 1 - para cada um dos fatores primos pk de sua decomposio.
pk

70

Licenciatura em Matemtica

EXEMPLO 4A:

Como 2 e 5 so os nicos primos na fatorao de 100, podemos fazer


1 1
1 4
f(100) = 100.1 - 1 - = 100. . = 40 .
2 5
2 5
EXEMPLO 4B:

Pela fatorao 72 = 23.32 , obtemos: f(72)

1 1
1 2
= 72.1 - 1 - = 72. . = 24 .
2 3
2 3

A
4

As informaes contidas neste tpico so, obviamente, apenas uma introduo s prin
cipais propriedades da funo f . No prximo tpico, estudaremos a funo m de Mbius.

Teoria dos nmeros

T
2

71

03
TPICO

A FUNO m DE MBIUS

OBJETIVOS

Reconhecer a funo m e algumas de suas


propriedades;
Definir quando um nmero livre de quadrados.

matemtico e astrnomo alemo August Ferdinand Mbius (1790 1868) de


senvolveu um trabalho de grande relevncia na Matemtica, especialmente
no campo da Geometria, com a famosa faixa (ou fita) de Mbius, e nas Vari

veis Complexas, com as transformaes de Mbius. Na Teoria dos Nmeros, deve-se a ele
o estudo sobre a funo que recebeu o seu nome e que apresentamos a seguir.
A funo de Mbius (l-se, aproximadamente, Mbius) alterada pela quantida
de de primos que aparecem na fatorao de um nmero, mas, ao contrrio das fun
es aritmticas anteriores, ela limitada.
Definio 5: Denotaremos pela letra grega m (mi) a funo que, para cada
nmero inteiro positivo n >1 , com fatorao em primos n = p1a1 . p2a2 ..... pkak , as
socia o nmero
(-1)k , se a1 = a2 = ... = ak = 1
m(n) =
.

0, caso contrrio
De modo a tornar m definida para todos os inteiros positivos e no alterar a sua
multiplicabilidade (ainda a ser demonstrada), definimos convenientemente m(1) = 1 .

72

Licenciatura em Matemtica

Assim, a funo m(n) vale 0 sempre que n possuir, em sua fatorao em primos,
um expoente maior ou igual a 2. Neste caso, n divisvel pelo quadrado de algum
primo. Mas para que seja divisvel pelo quadrado de um inteiro maior que 1, um
nmero deve tambm ser divisvel pelo quadrado de um primo. Assim, quando
m(n) 0 , dizemos que o inteiro n livre de quadrados.

A
4

EXEMPLO 1A:

Como 30 = 2.3.5 , vale m(30) = (-1) = -1 , pois 30 tem 3 primos distintos em


3

sua fatorao e nenhum dos expoentes maior que 1. Analogamente, a fatorao

T
3

10 = 2.5 leva a m(10) = (-1)2 = 1 .


EXEMPLO 1B:

Para qualquer primo p , tem-se m( p) = -1 .


EXEMPLO 1C:

A tabela abaixo lista os valores de m(n) para os 12 primeiros inteiros positivos.


n

10

11

12

m(n)

Proposio 4: A funo m multiplicativa.


Demonstrao: Para os nmeros inteiros positivos e relativamente primos
m e n , temos duas possibilidades a considerar. Se m e n forem livres de qua
drados, podemos escrever m = p1 p2 ... pk e n = q1q2 ...qt , onde os primos envolvi
dos so todos distintos, de modo que mn tambm ser livre de quadrados. Da,
temos mn = p1 p2 ... pk q1q2 ...qk e, portanto,

m(mn) = (-1)k +t = (-1)k (-1)t = m(m)m(n) .

Se para algum primo p , tivermos p2 |mn , isto , mn no for livre de quadra


dos, o mesmo deve acontecer com m ou n , j que eles so relativamente pri
mos. Assim, m(m) = 0 ou m(n) = 0 , da m(mn) = 0 = m(m)m(n) e obtemos o
mesmo resultado.

Nos tpicos anteriores, verificamos a multiplicabilidade de algumas funes de modo


a obter uma expresso simplificada para o clculo das imagens dessas funes quando
sabemos a fatorao em primos do nmero. Com tal fatorao, o valor obtido na funo
m encontrado diretamente, de modo que a proposio acima no ser usada para a
determinao de m , mas como resultado auxiliar para outros fatos, como o que segue.

Teoria dos nmeros

73

Proposio 5: Para qualquer nmero inteiro positivo n >1 , vale

m( d ) = 0 .
d|n

Demonstrao: Comecemos com o caso n = p k para algum primo p . Os di


visores de p k so 1, p, p2 ,..., p k , logo:

m(d ) = m(1) + m( p) + m( p ) + ... + m( p

A
4

d|p

)=

= 1 + (-1) + 0 + ... + 0 = 0.

T
3

Observe agora que, para m e n relativamente primos, procedemos de manei


ra semelhante ao que fizemos para provar que s multiplicativa. Acompanhe o
raciocnio, no qual empregamos o fato de m ser multiplicativa.

m(d ) = m(d d ) = m(d )m(d ) = m(d ). m(d ).

d1|m
d2|n

d|mn

1 2

d1|m d2|n

d1|m

d2|n

Dessa feita, para n = p1a1 . p2a2 ..... pkak , vale

m(d ) = m(d ). m(d )..... m(d ) .


d|n

d1|pa1

d2|pa2

dk|pak

Mas cada um destes fatores

nulo, de onde temos o resultado vlido para qualquer n >1 .


Com esta propriedade, encerramos o estudo inicial das propriedades da fun
o m . Alm de outros fatos interessantes sobre cada uma das funes estudadas
nesta aula, h uma srie de relaes relevantes entre elas. Tais relaes sero o
foco de nossa prxima aula.

74

Licenciatura em Matemtica

Aula 5

Funes aritmticas - parte II


Caro(a) aluno(a),

Na quinta aula de nosso curso de Teoria dos Nmeros, daremos prosseguimento ao


estudo das funes aritmticas, apresentando outras propriedades e, especialmente,
algumas relaes entre as funes aritmticas estudadas na aula 4, alm de apresentar uma funo especial, com domnio real, mas que desempenha papel relevante
na anlise dos nmeros inteiros, tanto no estudo das funes aritmticas quanto no
princpio das gavetas de Dirichlet, que ser tema da aula 6.

Objetivos:
Prosseguir com o estudo das funes aritmticas;
Apresentar a funo maior inteiro e algumas de suas propriedades.

75

01
TPICO

OUTRAS PROPRIEDADES
DAS FUNES ARITMTICAS

OBJETIVO

Apresentar algumas propriedades sobre as funes


aritmticas.

este tpico, continuaremos o estudo iniciado na


aula 4, na qual associamos, para cada inteiro po

ATENO!

sitivo n , os seguintes valores:

Observao 1: Na fatorao do
nmero 1 no aparece nenhum primo.
Assim, podemos usar a definio
ao lado tambm para o nmero 1,
fazendo, portanto, k = 0 .
Observao 2: A quantidade de
elementos do conjunto A pode ser
representada por #A , mas tambm h
as notaes n( A) , |A| ou card ( A) .

76

t (n) , que a quantidade de divisores positivos de n ,



por exemplo, t (15) = 4 . Assim,
t (n) = #{d + ; d |n} = 1
d|n

s (n) , que a soma dos divisores positivos de n , por



exemplo, s (15) = 24 . Assim,

Licenciatura em Matemtica

s (n ) = d
d|n

f(n) , que a quantidade de inteiros positivos menores que n e relativamente


primos com n , por exemplo, f(15) = 8 . Assim,
f(n) = #{m + ; m < n e (m, n) = 1}

m(n) , que 0, se n no for livre de quadrados, e (-1)k , se k for a quantidade de


primos distintos que aparecem na fatorao de n , por exemplo, m(15) = 1 . Assim:

A
5

(-1)k , se n = p1 p2 ... pk
m(n) =
0, caso contrrio
Vimos

que

todas

essas

funes

so

multiplicativas,

em

T
1

particular

t (1) = s (1) = f(1) = m(1) = 1 . Vejamos agora outras propriedades interessantes sobre
as funes aritmticas, que sero demonstradas tomando em conta o fato de que se
n
d |n , ento a = inteiro e a |n .
d

Proposio 1: Para qualquer inteiro positivo n , vale

d = n

t ( n )/2

d|n

Demonstrao: Aqui basta verificar que sempre que d |n , ento a =

n
inteiro
d

e a |n . Assim, se
n
Q = d , vale, igualmente, Q = . Da, multiplicando as duas igualdades, te
d|n
d|n d
n
2
mos Q = d = n . No segundo membro, temos fatores todos iguais a n e tan
d d|n
d|n

tos quantos forem os divisores de n , ou seja, h t (n) fatores. Assim, Q 2 = n t ( n ) , e


como Q > 0 , basta extrair a raiz quadrada nos dois membros da igualdade para ob
ter o resultado.
Na expresso acima, poderamos pensar que o resultado do segundo membro re
sultaria em um nmero no inteiro, caso t (n) fosse mpar. Mas t (n) obtido pelo
produto dos consecutivos dos expoentes da fatorao em primos de n , de modo que
somente resultar t (n) mpar se todos os expoentes da fatorao forem pares e, as
sim, n quadrado perfeito e n t ( n )/2 inteiro.

Proposio 2: Para todo inteiro positivo n , vale

f( d ) = n .
d|n

Teoria dos nmeros

77

Demonstrao: Comecemos separando os nmeros do conjunto A = {1,2,..., n}


em subconjuntos de acordo com o segue:

A1 = {m A;(m, n) = 1} ;

A2 = {m A;(m, n) = 2} ;

...

A
5

An = {m A;(m, n) = n} .

Observe que no conjunto Ak = {m A;(m, n) = k} ser vazio se k no for


um divisor de n . Assim, teremos t (n) conjuntos no vazios e An = {n} .

T
1

Agora analisemos a quantidade de elementos de Ad para cada divisor de n . Ora,


m
n
para que m Ad necessrio que (m, n) = d , mas isto significa que
e
so
d
d
m n
inteiros sem divisores prprios comuns, ou seja, ( , ) = 1 . Dessa forma, cada con
d d
m
junto Ad possui f elementos. Como os subconjuntos Ad assim formados so
d
disjuntos, a quantidade de elementos da unio de todos vale # Ad = f(n / d ) .
d|n

Mas

d|n

= A , que possui n elementos, de modo que temos a igualdade

d|n

f(n / d ) = n . Mas, de novo usando o argumento de que quando d

percorre os

d|n

divisores de n , os valores de n / d tambm percorrem esses mesmos divisores, sem


repetio, de modo que

f(n / d ) = f(d ) , da vale f(d ) = n .


d|n

d|n

d|n

Antes de prosseguir com outras propriedades das funes aritmticas, vamos estu
dar a funo maior inteiro, que, embora no seja uma funo com domnio no conjun
to dos nmeros inteiros, tem significativa importncia na Teoria dos Nmeros, como
veremos a seguir.

78

Licenciatura em Matemtica

02
TPICO

A FUNO MAIOR INTEIRO

OBJETIVO

Apresentar a funo maior inteiro.

o estudo das funes aritmticas, uma nova funo surge para desempe
nhar papel relevante, ainda que no seja, ela prpria, aritmtica no sen
tido de ter domnio no conjunto dos inteiros positivos. Com os conheci

mentos elementares sobre o conjunto dos nmeros reais, podemos trabalhar com a
seguinte definio, que ser fundamentada no seguinte argumento
Definio 1: Dado o nmero real x, representamos por x o maior inteiro que
menor ou igual a x, equivalentemente, escrevemos: x = max{n ; n x} .
A funo f : dada por f (x ) = x chamada de funo maior inteiro.
O valor x tambm pode ser chamado de parte inteira de x.

80

Licenciatura em Matemtica

EXEMPLO 1A:

Para qualquer nmero inteiro n , vale diretamente da definio que n = n , de


modo que podemos concluir daqui que a funo maior inteiro sobrejetiva.
EXEMPLO 1B:

Valem as igualdades 2 = 1 , p = 3 , e = 2 e -2,34 = -3 .

A
5

EXEMPLO 1C:

A soluo (real) da equao x = 4 o intervalo [4;5) , de modo que podemos


concluir daqui que a funo maior inteiro no injetiva, pois 4 = 4,1

T
2

EXEMPLO 1D:

Suponha que o nmero inteiro positivo M , quando escrito na base decimal, te


nha a forma M = (ak ...a1a0 )10 = ak .10 k + ... + a1 .10 + a0 , com ak 0 , ou seja, tenha
k + 1 algarismos. Temos aqui as desigualdades 10 k M < 10 k +1 . Aplicando o loga
ritmo decimal, obtemos k log M < k + 1 , da log M = k e, assim, a quantidade
de algarismos de M na base decimal log M + 1 . Mais geralmente, podemos veri
ficar que a quantidade de algarismos usados para representar o nmero inteiro posi
tivo M na base B > 1 ser igual a logB M + 1 .
EXEMPLO 1E:

Dados os inteiros positivos a e b , podemos, pelo algoritmo da diviso, encontrar


inteiros q e r tais que a = bq + r , com 0 r < b . Assim, temos:
a bq r
r
= + = q + , de conclumos
b
b b
b

a
r
r
= q + = q , pois 0 < 1 .
b

b
b

Veja que para cada nmero inteiro n , a funo maior inteiro constante e vale
n dentro do intervalo [n; n + 1) . Assim, o grfico da funo maior inteiro formado
de segmentos de reta horizontais, de comprimento 1, de acordo com o que segue:

Figura 1: Representao da funo maior inteiro

Teoria dos nmeros

81

A respeito da funo maior inteiro, valem as seguintes propriedades, cujas de


monstraes so deixadas como exerccio.
(1) x + n = x + n , para qualquer real x e qualquer inteiro positivo n .
(2) x - 1 < x x , para qualquer real x .
(3) Se x no um nmero inteiro, ento -x = - x - 1 .
(4) x = x , para qualquer real x .

A
5

(5) Se x < y , ento x y .


(6) x + y x + y x + y + 1 , para quaisquer reais x e y .

T
2

Observao 1: A propriedade (4) pode ser reescrita nos seguintes termos:


se f (x ) = x , ento f ( f (x )) = f (x ) . Uma funo que satisfaz a propriedade
f ( f (x )) = f (x ) para qualquer elemento do domnio dita idempotente.
Observao 2: A propriedade (5) pode ser reescrita nos seguintes termos: se
f (x ) = x , ento x < y f (x ) f ( y ) . Uma funo que satisfaz a propriedade
x < y f (x ) f ( y ) para quaisquer elementos do domnio dita montona no decrescente. Se x < y f (x ) f ( y ) , ento f montona no crescente.
EXEMPLO 2A:

Sabendo que x = 4 , determine os possveis valores de 3x .


Soluo:

Observe que x = 4 se verifica para todos os valores x [4;5) , isto ,


x = 4 4 x < 5 , de onde obtemos que 12 3x < 15 , da os possveis valores
para 3x so 12, 13 e 14.
EXEMPLO 2B:

Se a e b so nmeros reais tais que a = 6 e b = -3 , encontre o menor valor


possvel para 2a + 3b .
Soluo:

GUARDE BEM ISSO!


Podemos usar a funo maior
inteiro para estender as funes
aritmticas estudadas na aula 4 para
todos os nmeros reais, definindo,
por exemplo, f( x ) = f ( x ) para
qualquer nmero real, embora
esta extenso no acrescente fatos
significativos nossa teoria.

82

Inicialmente

temos

a = 6 6 a < 7
b = -3 -3 b < -2 .

que

Da, multiplicando por 2 e 3, respectivamente, ob


temos 12 2a < 14 e -9 3b < -6 e somando es
tes

resultados,

conclumos

que

3 2a + 3b < 8 .

Assim, o menor valor possvel para 2a + 3b 3 e o


maior valor possvel 7.

Licenciatura em Matemtica

Tambm interessante perceber que, dados os inteiros positivos a e b , a quan


a
tidade de nmeros menores ou iguais a a que so divisveis por b .
b
Vistas as propriedades iniciais a respeito da funo maior inteiro, vamos voltar s
funes aritmticas e mais algumas relaes entre elas.

A
5
T
2

Teoria dos nmeros

83

03
TPICO

OUTRAS RELAES

OBJETIVO

Estudar outras
aritmticas;

relaes

entre

as

funes

Relacionar as funes aritmticas e a funo


maior inteiro.

ejamos agora algumas outras relaes entre a Teoria dos Nmeros e a fun
o maior inteiro.

Proposio 3: Se n um nmero inteiro positivo e p primo, ento pa |n !


n n
n
se, e somente se, a + 2 + ... + k , onde k o maior inteiro pa
p p
p


ra o qual p k n . Alternativamente, a maior potncia de p que divide n !
n n
n
a = + 2 + ... + k .


p p
p
Demonstrao: Observe inicialmente que poderamos ter acrescentado
n n
n
qualquer potncia de p soma + 2 + ... + k , pois se p k > n , vale
p p
p


n
= 0 . Denote por ai a quantidade de nmeros menores ou iguais a n que
pk

so divisveis por pi .
Como n ! = 1.2.....n , ento teremos que a = a1 + a2 + ... + ak , mas os nmeros
n
n
naturais que so divisveis por pi so pi ,2. pi ,3. pi ,..., i . pi . Assim, ai = i
p
p


n n
n
e a = + 2 + ... + k , como desejado.


p p
p

84

Licenciatura em Matemtica

EXEMPLO 1A:

Se quisermos saber qual a maior potncia de 2 que di


vide 21! fazemos

21 21 21 21
= + + + =
2 4 8 16
= 10 + 5 + 2 + 1 = 18.

Assim, 218 |21! e, logo, 2k |21! , para qualquer


k = 0,1,...,18 .

ATENO!
A proposio acima apenas conta
quantos nmeros de 1 a n so
divisveis por p , depois conta
quantos so divisveis por p2 e assim
sucessivamente.

T
3

EXEMPLO 1B:

19
A maior potncia de 5 que divide 19! = 3 . De
5
fato, dos nmeros inteiros de 1 a 19, apenas 5, 10 e 15 so mltiplos de 5, e nenhum
deles mltiplo de 25.
Adiante, veremos uma relao entre as funes f e m .

n
Proposio 4: Para qualquer inteiro positivo n , vale f(n) = m(d ). .
d
d|n

1
= 1 , se m e n fo
Demonstrao: Veja que para m {1,2,..., n} , temos
(m, n )

1
= 0 , caso contrrio. Assim, obtemos uma
rem relativamente primos e
(m, n )

n
1
nova maneira de obter o valor f(n) , atravs da igualdade f(n) =
.

k =1 ( k , n )
Vimos no final da aula 4 que m(d ) = 0 para qualquer inteiro positivo n > 1 .
d|n

Uma vez que

m( d ) = 1 ,
d|1

igualdade

m(d ) = n ,

podemos sintetizar as duas informaes com a

vlida para qualquer inteiro positivo n . Logo

d|n

m(d ) = (k, n) .

d|( k ,n )

n
1
Fazendo uso dessa ltima igualdade em f(n) =
, obtemos:

k =1 ( k , n )
n
f ( n ) = m( d ) .
k =1 d|( k ,n )

Agora, se d |(n, k ) , verdade tambm que d |n e d |k , de modo que podemos


ainda escrever:

Teoria dos nmeros

A
5

85

f(n) = m(d ) .
k =1 d|k
d|n

Sabemos que d |k equivale a existir um inteiro q tal que k = qd , mas para


n
1 k n , temos 1 q . Da, obtemos:
d
n/d
n/d

f(n) = m(d ) = m(d ). 1 = m(d ). , o que completa a demons


d

d|n
d|n q=1
d|n
q=1
trao.

A
5
T
3

EXEMPLO 2:

Com a expresso obtida anteriormente, podemos encontrar f(n) de outra manei


20
ra. Veja que f(20) = m(d ). . Como os divisores inteiros positivos de 20 so 1,
d
d|n
2, 4, 5, 10 e 20, podemos fazer:

f(20) = m(1).

20
20
20
20
20
20
+ m(2). + m(4). + m(5). + m(10). + m(20).
1
2
4
5
10
20

Mas 4 e 20 no so livres de quadrados, logo m(4) = m(20) = 0 . Alm disso, 2 e 5


so primos, de onde temos m(2) = m(5) = -1 e, pela definio, temos m(1) = m(10) = 1 .
Assim:

f(20)

20
20
20
20
20
20
+ (-1). + 0. + (-1). + 1. + 0. =
1
2
4
5
10
20
= 20 - 10 + 0 - 4 + 2 + 0 = 8.
= 1.

Com este tpico, encerramos nossa aula. Para melhor compreenso das ideias so
bre as funes aritmticas e da funo maior inteiro, recomenda-se uma recapitulao
dos exemplos, com a troca dos nmeros para melhor fixao dos conceitos e relaes.
Como peas fundamentais da nossa teoria, vez por outra, revisitaremos as fun
es aritmticas, nesta ou em outras disciplinas. Na prxima aula, tambm voltare
mos a aplicar a funo maior inteiro.

86

Licenciatura em Matemtica

Aula 6

O princpio das gavetas


Caro(a) aluno(a),

Em nossa quarta aula, estudaremos o princpio das gavetas, o qual afirma que, se
voc tiver n + 1 objetos em n gavetas, pelo menos uma dela conter mais de um
objeto. uma afirmao simples e pode at ser considerada bvia, mas traz consequncias de grande relevncia.

Objetivos:
Apresentar o Princpio de Dirichlet e problemas nos quais o princpio pode ser aplicado;
Destacar resultados do Princpio de Dirichlet na Teoria dos Nmeros.

87

01
TPICO

INTRODUO

OBJETIVOS

Apresentar exemplos iniciais;


Enunciar formalmente o princpio.

matemtico Johann Dirichlet (1805 - 1859) provou um caso particular ( n = 5 )


do teorema de Fermat (veja comentrio no incio do tpico 2 da aula 3). Com
seus estudos, ele proporcionou outras inmeras contribuies para a Mate

mtica e a Estatstica, dentre elas o princpio que nortear nossa aula o qual pode ser
colocado nos seguintes termos: se n + 1 pombos forem colocados em n gaiolas, pelo
menos uma das gaiolas conter pelo menos dois pombos. Obviamente o resultado con
tinua valendo para qualquer quantidade de pombos que seja superior quantidade de
gaiolas. Descrevendo mais tecnicamente, podemos enunciar da seguinte forma:
Princpio de Dirichlet: Se A e B so conjuntos finitos e A tem mais elementos que
B , ento no pode haver uma funo injetiva f : A B , isto , haver elementos
distintos x, y A tais que f (x ) = f ( y ) .

88

Licenciatura em Matemtica

Observe que, se P for o conjunto de n + 1 pombos e G o conjunto de n gaiolas,


a funo que associa cada pombo sua gaiola no pode ser injetiva, de onde obte
mos, naturalmente, o mesmo resultado, isto , que dois pombos (pelo menos) devem
ocupar a mesma gaiola.
EXEMPLO 1A:

Em uma turma de 13 pessoas, necessariamente duas delas fazem aniversrio no

A
6

mesmo ms. Para verificar isso, podemos pensar nas pessoas como os objetos e os
meses como as gavetas. Ao distribuir 13 pessoas nos doze meses do ano, necessa

T
1

riamente um dos meses deveria conter pelo menos duas pessoas.


EXEMPLO 1B:

Em uma lista de 6 nmeros inteiros quaisquer, pelo menos dois deles deixam o
mesmo resto na diviso por 5, pois os restos possveis so 0, 1, 2, 3 ou 4. Nesse ca
so, temos seis nmeros para distribuir em 5 possveis restos. Usando a notao de
funo, basta considerar a funo que associa cada um dos seis nmeros dados ao
seu resto na diviso por 5.
EXEMPLO 1C:

Em um pargrafo com 27 palavras, pelo menos duas delas comearam com a mes
ma letra.
EXEMPLO 1D:

No interior de um quadrado de lado 2 m, so marcados cinco pontos. Se dividirmos


o quadrado em quatro quadrados menores de lado 1 m e consequentemente de dia
gonal

2 @ 1,41 m podemos associar cada ponto ao quadrado menor que o contm.

Pelo princpio de Dirichlet, haver pelo menos dois pontos no mesmo quadrado, mas a
distncia mxima entre dois pontos de um quadrado a medida da sua diagonal, assim
haver pelo menos dois pontos que estaro a uma distncia inferior a

2 m.

com a ideia do princpio de Dirichlet e com algumas generalizaes que vamos


trabalhar do decorrer desta aula. A obviedade do princpio indica a simplicidade das
demonstraes, sendo que a parte difcil (que deixa de ser difcil com um pouco de pr
tica) identificar, dentro do problema dado, o que so os objetos e as gavetas ou,
equivalentemente, entre quais conjuntos vamos definir a funo que no ser injetiva.
EXEMPLO 2A:

Mostre que h um mltiplo de 19 cuja representao decimal contenha apenas


algarismos 0 e 1.

Teoria dos nmeros

89

Soluo:

Considere a sequncia dos nmeros formados apenas por algarismos 1:


1, 11, 111, ....
Se da lista considerarmos os 20 primeiros nmeros, necessariamente dois de
les devero deixar o mesmo resto na diviso por 19. Para tanto, basta considerar a
funo que associa cada nmero ao resto da sua diviso por 19, que so apenas 19

A
6

possibilidades. Sabemos que, se dois nmeros deixam o mesmo resto na diviso por
19, ento a subtrao entre eles deixa resto 0, ou seja, mltiplo de 19 (lembre que
a b(mod19) equivale a 19|a - b ). Mas a subtrao de dois dos nmeros conside

T
1

rados gera um nmero formado apenas pelos algarismos 1 e 0, como desejado.


EXEMPLO 2B:

Mais geralmente, dado o inteiro positivo n e um algarismo a {1,2,...,9} , pode


mos encontrar um mltiplo de n cuja representao decimal contenha apenas os al
garismos 0 e a . Para tanto, basta considerar os n + 1 primeiros termos da sequncia
dos nmeros formados pela justaposio de algarismos a , isto , a, aa,..., aa...a ,
com o ltimo nmero tendo n + 1 algarismos todos iguais a a e aplicar o raciocnio
descrito anteriormente.
Alternativamente, podemos exprimir a ideia contida no princpio das gavetas
tambm dizendo que, se tivermos n gavetas, a quantidade mnima de objetos a se
rem postos nas gavetas de modo a assegurar que haja pelo menos uma gaveta com
pelo menos dois objetos n + 1 .
EXEMPLO 3A:

Em uma urna, h 7 bolas pretas, 5 bolas brancas e 10 bolas azuis. Qual a quanti
dade mnima de bolas que devem ser retiradas s cegas para se garantir que h duas
bolas da mesma cor?
Soluo:

Aqui temos o conjunto das bolas retiradas e para cada uma delas podemos associar
uma cor. Como h trs cores possveis, devemos tirar 4 bolas para garantir que essa fun
o no seja injetiva, ou seja, para termos certeza de que duas bolas tenham a mesma cor.
EXEMPLO 3B:

O conjunto S formado por todos os inteiros que so relativamente primos com


18. Determine a quantidade mnima de elementos de S que devem ser tomados para
que haja dois deles cuja diferena divisvel por 18.

90

Licenciatura em Matemtica

Soluo:

Para que a diferena entre dois nmeros seja um mltiplo de 18, eles devem dei
xar o mesmo resto na diviso por 18 e os restos possveis nessa diviso so 0, 1, 2,
..., 17, de onde poderamos pensar que a resposta seria no mnimo 19 nmeros.
Entretanto devemos atentar para o fato de que no estamos considerando nmeros
inteiros quaisquer, mas apenas aqueles que so relativamente primos com 18, de

A
6

modo que o conjunto de gavetas no contm todos os resduos possveis (no


um sistema completo), mas apenas aqueles que so primos com 18 (um sistema re
duzido). Assim, temos f(18) possibilidades para esses resduos. Como f(18) = 6 , a

T
1

quantidade mnima de elementos de S que devem ser tomados para que se tenha a
certeza de haver dois cuja diferena seja um mltiplo de 18 7.
Seguindo as ideias contidas nos exemplos deste tpico, podemos construir uma
srie de outros. Um exerccio interessante colecionar esses exemplos e discutir co
mo o princpio de Dirichlet pode ser aplicado. Adiante, discutiremos como podemos
generalizar os raciocnios envolvidos.

Teoria dos nmeros

91

02
TPICO

GENERALIZAO DO
PRINCPIO DAS GAVETAS

OBJETIVOS

Estudar situaes generalizadas do problema das


gavetas;
Aplicar a funo maior inteiro nas solues.

ontinuando com o princpio discutido no tpico anterior, podemos pensar


que se nk + 1 objetos forem colocados em n gavetas, ento pelo menos uma
das gavetas conter k + 1 objetos. Esse fato pode ser demonstrado se agru

parmos os nk + 1 em n grupos de k objetos e mais um objeto avulso. Temos a n + 1


grupos de objetos para serem colocados em n gavetas, de onde pelo menos uma ga
veta dever conter pelo menos dois desses grupos, ou seja, no mnimo k + 1 objetos.

Princpio de Dirichlet (generalizao) Se A e B so conjuntos com nk + 1 (ou


qualquer quantidade superior a nk ) e n elementos, respectivamente, e f : A B
uma funo, ento haver um elemento de B que imagem de pelo menos k + 1
elementos de A .
EXEMPLO 1A:

Em um grupo de 36 pessoas, h pelo menos seis delas que fazem aniversrio no

92

Licenciatura em Matemtica

mesmo dia da semana em 2010. Para tanto, basta verificar que os dias da semana (as
gavetas) so 7 e as pessoas (objetos) so 36 = 5.7 + 1, de modo que o princpio pode
ser aplicado e haver pelo menos um dia correspondente a 6 pessoas.
EXEMPLO 1B:

Mostre que de um conjunto de 41 nmeros distintos, podemos escolher cinco


cuja soma um mltiplo de 5.

A
6

Soluo:

Se somarmos cinco nmeros congruentes mdulo 5, teremos necessaria

T
2

mente um mltiplo de 5 como resultado. Observe que, se ai k(mod5) pa


ra qualquer elemento do conjunto {a1 , a2 ,...a5 } , ento a soma das congrun
cias levar a a1 + a2 + ... + a5 5k(mod5) , mas 5k um mltiplo de 5, da
a1 + a2 + ... + a5 0(mod5) , sendo a soma tambm um mltiplo de 5. Agora temos
que garantir a existncia de pelo menos cinco nmeros que deixam o mesmo resto
na diviso por 5. Os restos possveis so 5 (as gavetas) e os nmeros (objetos) so 21
= 4.5 + 1. Usando o princpio para n = 5 e k = 4 , garantimos que h pelo menos 5
nmeros que deixam o mesmo resto na diviso por 5. Analogamente, podemos esta
belecer a funo que associa cada nmero ao resto na diviso por 5 e usar a formu
lao em termos de funo para o princpio de Dirichlet e obter o mesmo resultado.
Assim como feito no tpico anterior, tambm podemos obter a quantidade mni
ma de objetos a serem considerados para que tenhamos pelo menos uma gaveta com
uma quantidade pr-estabelecida de objetos, depois da distribuio.
EXEMPLO 2A:

Cada um dos dez alunos de uma turma recebe uma cartela com quatro nmeros in
teiros positivos, distintos e menores ou iguais a 40, de modo que cada nmero aparece
em apenas uma cartela. Sero sorteados sucessivamente os nmeros de uma urna, um
por um, at que um dos alunos preencha sua cartela, caso em que ser determinado
vencedor. Qual a quantidade mxima de nmeros sorteados nessa atividade?
Soluo:

Obviamente, pode acontecer de os quatro primeiros nmeros sorteados estarem


na cartela de apenas um aluno, mas o sorteio pode continuar. Aqui queremos saber
a quantidade mnima de nmeros que devem ser sorteados para que se tenha certe
za de que pelo menos um deles tenha 4 nmeros. Se pensarmos nos nmeros como
os objetos e nas cartelas como as gavetas, queremos que cada uma delas tenha 4, ou
seja, 3 + 1 objetos. Como so 10 cartelas ao todo, podemos fazer n = 10 e k = 3 , as
sim com a quantidade de nk + 1 = 31 nmeros sorteados, teremos certeza de ter um

Teoria dos nmeros

93

aluno vencedor, ento no chegaremos a sortear o 32 nmero.


EXEMPLO 2B:

Determine a quantidade mnima de pessoas que devem ser tomadas para que se
possa formar um grupo de cinco pessoas com aniversrio no mesmo ms.
Soluo:

A
6

Aqui podemos pensar em n = 12 (os meses como as gavetas) e k + 1 = 7 (a quan


tidade mnima de objetos que queremos garantir em pelo menos uma das gavetas).
Assim, para k = 6, n = 12 , temos que em um grupo de nk + 1 = 73 pessoas, h ne

T
2

cessariamente um grupo de cinco com aniversrio no mesmo ms.


Para o que segue da nossa teoria, faremos uso da funo maior inteiro e de suas
propriedades estudadas na aula 5. Lembremos que, dado o nmero real x , repre
sentamos por x o maior inteiro que menor ou igual a x . Por exemplo: 3,78 = 3 .
A proposio a seguir uma reformulao do princpio de Dirichlet, verso gene
ralizada, acompanhada de uma demonstrao.
Proposio 1: Se colocarmos k objetos em n gavetas, ento pelo menos uma gaveta

k - 1
+ 1 objetos.
n

conter pelo menos

Demonstrao: Da definio da funo maior inteiro, vale x x , para qualquer x


real, logo

k - 1 k - 1

n
n

k - 1
objetos. Assim, teremos, no
n

Supondo que cada gaveta contenha no mximo

k - 1
objetos. Ento:
n

mximo, n.

k - 1
k -1
n.
= k -1 < k
n.
n
n

Assim, a quantidade de objetos nunca chegaria a k , o que uma contradio.


EXEMPLO 3A:

Se observarmos os meses de nascimento de um grupo de 50 pessoas, temos


k = 50 e n = 12 , assim podemos garantir que haver pelo menos um ms com
50 - 1

+ 1 = 4 + 1 = 5 pessoas, ou seja, podemos concluir que, de um grupo de 50


12

94

Licenciatura em Matemtica

pessoas, poderemos certamente escolher cinco que fazem aniversrio no mesmo ms.
EXEMPLO 3B:

De um conjunto de 100 nmeros, podemos garantir que h pelo menos 8 nme


ros que deixam o mesmo resto na diviso por 13. Basta usar aqui k = 100 e n = 13 .
Com essa formulao geral do princpio de Dirichlet, encerramos este tpico. No
prximo, veremos como aplic-lo de outras maneiras tambm interessantes.

A
6
T
2

Teoria dos nmeros

95

03
TPICO

EXEMPLOS GERAIS

OBJETIVOS

Estudar situaes generalizadas do problema das


gavetas;
Aplicar a funo maior inteiro nas solues.

ara finalizar nossa aula, vejamos como princpio de Dirichlet pode ser apli
cado em outras situaes.
EXEMPLO 1:

Se o plano for pintado de verde e azul, prove que haver dois pontos de mesma

cor cuja distncia exatamente 1 metro.


Soluo:

Basta construir um tringulo equiltero de lado 1 m. Os vrtices do tringulo (3) so


em quantidade maior que o de cores possveis (2), logo haver dois vrtices de mesma cor.
EXEMPLO 2:

Os nmeros inteiros de 1 a 10 so escritos em um crculo, em qualquer ordem.

96

Licenciatura em Matemtica

Mostre que h trs nmeros adjacentes cuja soma maior ou igual a 17.
Soluo:

Podemos formar 10 sequncias diferentes de trs nmeros adjacentes, de modo que,


se somarmos todas as sequncias possveis, cada nmeros aparecer 3 vezes. A soma to
tal ser, portanto, 3.(1 + 2 + ... + 10) = 3.55 = 165. Se todas as somas de trs nmeros
adjacentes forem menores que 17, a soma total seria, no mximo, 10.16 = 160, o que no

A
6

verdade. Logo uma das sequncias, pelo menos, deve ter soma maior ou igual a 17.
EXEMPLO 3:

T
3

Prove que h duas potncias de 3, distintas, cuja diferena divisvel por 2011.
Soluo:

Considere o conjunto formado pelas 2012 primeiras potncias de 3, isto :


31 ,32 ,33 ,...,32012 .
Como o nmero de elementos do conjunto maior que a quantidade de possveis
restos na diviso por 2011, haver dois deles congruentes mdulo 2011, ou seja, dois
deles deixam mesmo resto na diviso por 2011. Isto , h inteiros positivos m, n dis
tintos tais que 3m 3n (mod2011) , ou seja, 2011|3m - 3n .
EXEMPLO 4:

Qual a quantidade mnima de brasileiros que devemos escolher para garantir que se possa
formar um grupo com 6 pessoas que nasceram na mesma unidade da federao (total de 27)?
Soluo:

Aqui usamos a formulao geral do princpio de Dirichlet, para n = 27 e k = 5 ,


de modo que o mnimo de pessoas a serem selecionada para se ter certeza da proprie
dade desejada 27.5 + 1 = 136 .
Com essas ideias, encerramos a nossa aula. A sugesto sempre buscar novos
exemplos para complementar a teoria, bem como novos problemas que possam ser
resolvidos como princpio apresentado aqui, sempre com o cuidado de identificar os
objetos (ou pombos) e as gavetas (ou casas).

Teoria dos nmeros

97

Aula 7

Resduos quadrticos
Caro(a) aluno(a),

Dando prosseguimento ao estudo da Teoria dos Nmeros, em nossa stima aula,


continuaremos analisando as propriedades das congruncias e verificando as condies que assegurem a existncia de solues para equaes de tipo especfico.

Objetivos:
Estudar os resduos quadrticos e o smbolo de Legendre.
Apresentar a lei da reciprocidade quadrtica.

99

01

RESDUOS
QUADRTICOS

TPICO

OBJETIVOS

Definir os resduos quadrticos


Obter a quantidade de resduos quadrticos

qui sero estudadas as equaes do tipo x 2 a(mod n) , o que pode ser


interpretado como o problema de encontrar razes quadradas mdulo n
de um nmero inteiro positivo a . Pelo que sabemos sobre sistemas com

pletos de resduos, as solues da equao dada s precisam ser procuradas no con


junto {0,1,2,..., n - 1} .
EXEMPLO 1:

Encontre todas as solues da equao x 2 1(mod5) .


Soluo:

Analisemos os casos:
se x 0(mod5) , ento x 2 0(mod5) e, assim, x 2 1(mod5)
se x 1(mod5) , ento x 2 1(mod5) e, assim, os nmeros da forma x 5k + 1 ,
com k inteiro so solues para o problema
se x 2(mod5) , ento x 2 4(mod5) e, assim, x 2 1(mod5)
se x 3(mod5) , ento x 2 4(mod5) e, assim, x 2 1(mod5)
se x 4(mod5) , ento x 2 1(mod5) e, assim, os nmeros da forma x 5k + 4 ,
com k inteiro so solues para o problema.

100

Licenciatura em Matemtica

Portanto, as solues para a equao dada so os nmeros da forma x 5k + 1


ou x 5k + 4 , com k inteiro. Uma vez que 4 -1(mod5) , podemos escrever a so
luo geral x = 5k 1 .
Pelo que foi visto no exemplo anterior, para qualquer nmero inteiro n , va
le n2 0(mod5) , n2 1(mod5) ou n2 4(mod5) e as equaes x 2 2(mod5) e

A
7

x 2 3(mod5) no possuem solues. Motivados por essa situao, daremos, em re


lao ao conjunto dos possveis restos na diviso por 5, um destaque aos nmeros 1

T
1

e 4, que sero chamados de resduos quadrticos mdulo 5.


Definio 1: Dados os nmeros inteiros positivos a e n , relativamente primos,
dizemos que a um resduo quadrtico mdulo n se a equao x 2 a(mod n)
possuir solues.
EXEMPLO 2A:

Os nmeros 1 e 4 so resduos quadrticos mdulo 5. Tambm 21 resduo qua


drtico mdulo 5.
EXEMPLO 2B:

Veja inicialmente que


x 0(mod6) x 2 0(mod6)

x 1(mod6) x 2 1(mod6)

x 2(mod6) x 2 4(mod6)

x 3(mod6) x 2 3(mod6)

x 4(mod6) x 2 4(mod6)

x 5(mod6) x 2 1(mod6)

Assim, somente para a = 0,1,3,4 a equao x 2 a(mod6) possui soluo intei


ra, mas 3 e 4 no so relativamente primos com 6, e 0 no positivo, de modo que 1
o nico resduo quadrtico mdulo 6 (a menos de congruncia mdulo 6).

Proposio 1: Se a um resduo quadrtico mdulo m , e b um res


duo quadrtico mdulo n , com m, n relativamente primos, ento a equao
z 2 ab(mod mn) possui soluo.

Demonstrao: Pela definio, temos que existem inteiros x, y tais que


x 2 a(mod m) e y 2 b(mod n)

Teoria dos nmeros

101

Uma vez que (m, n) = 1 , vale x 2 y 2 ab(mod mn) , ou seja, a equao


z 2 ab(mod mn) possui, pelo menos, a soluo xy .
Pelo exposto na proposio acima, podemos procurar os resduos quadrticos
mdulo n , procurando os resduos quadrticos referentes aos divisores de n . As
sim, se n for composto, podemos reduzir o problema para os seus divisores primos.

A
7

Portanto, a partir de agora, consideraremos apenas as equaes x 2 a(mod p) , com

T
1

relativamente primos.

p primo, caso em que tambm no nos ateremos condio de serem relativamen


te primos, pois todos os nmeros inteiros positivos menores que p , primo, so-lhe
Alm disso, o caso p = 2 resolvido diretamente, com todo nmero mpar
sendo um resduo quadrtico mdulo 2. Ento, por toda esta aula, p denotar
um primo mpar.
EXEMPLO 3:

Encontre todos os resduos quadrticos mdulo 7.


Soluo:

Um sistema completo de resduos mdulo 7 {0, 1, 2, 3, 4, 5, 6} e podemos ana


lisar os valores dos restos da diviso de x 2 por 7 em cada caso. Mais simplesmente,
podemos considerar o sistema completo {0, 1, 2, 3}.
se x 1(mod7) , ento x 2 1(mod7) , ou seja, 1 um resduo quadrtico
mdulo 7
se x 2(mod7) , ento x 2 4(mod7) , ou seja, 4 um resduo quadrtico
mdulo 7
se x 3(mod7) , ento x 2 2(mod7) , ou seja, 2 um resduo quadrtico
mdulo 7
Logo, os resduos quadrticos mdulo 7 (menores que 7) so 1, 2 e 4, e os resduos
no quadrticos so 3, 5 e 6.
Para finalizar o tpico, vamos a duas proposies que levam determinao da
quantidade de resduos quadrticos de um determinado nmero primo, motivado
p -1
pelo que se pode observar nos exemplos dados e no fato de que {0, 1, 2,
}
2
um sistema completo de resduos mdulo p , quando p mpar.
Lema: Se p primo mpar e a um inteiro positivo tal que a um resduo
quadrtico mdulo p , ento a equao x 2 a(mod p) possui exatamente duas
solues incongruentes mdulo p .

102

Licenciatura em Matemtica

Demonstrao: Primeiramente, veja que se x soluo para a equao dada, en


to p - x tambm soluo, pois ( p - x )2 = p2 - 2 px + x 2 e como p2 - 2 px
um mltiplo de p , temos ( p - x )2 x 2 (mod a ) . Alm disso, x e p - x so incon
gruentes mdulo p , porque, se valesse x p - x(mod p) , teramos 2x p(mod p)
, ou seja, p|2x , o que levaria a p|x e, consequentemente, x 2 0(mod p) , o
que no acontece. Com isso, obtemos que a equao possui pelo menos duas so

A
7

lues incongruentes mdulo p . Devemos mostrar tambm que qualquer ou


tra soluo para o problema congruente a x ou a p - x . Seja ento y tal que

T
1

y 2 a(mod p) , isto , y 2 x 2 (mod p) . Daqui conclumos que y 2 - x 2 0(mod p)


, isto , (x + y )(x - y ) 0(mod p) . Como p primo, temos que y + x 0(mod p)
ou y - x 0(mod p) , o que equivalente a y -x(mod p) ou y x(mod p) , mas
como -x p - x(mod p) , obtemos que ou y p - x(mod p) ou y x(mod p) , e
terminamos a demonstrao.
EXEMPLO 4A:

Uma vez que 52 3(mod11) , temos que 3 um resduo quadrtico mdulo 11 e as


nicas solues (a menos de congruncia mdulo 11) para a equao x 2 3(mod11)
so 5 e 6 (pois 6 = 11 5 )

Teorema: H exatamente
congruentes mdulo p .

p -1
resduos quadrticos mdulo p que so in
2

Demonstrao: Como {0,1,..., p - 1} um sistema completo de resduos m


dulo p , os resduos quadrticos mdulo p so gerados se obtivermos os res
tos de x 2 por p , para x = 1,2,..., p - 1 . Observe, porm, como visto no lema,
que x 2 e ( p - x )2 deixam o mesmo resto na diviso por p , logo todos os res
2
p - 1
duos quadrticos mdulo de p so os restos na diviso de 12 ,22 ,...,
.
2
p -1
resduos quadrticos mdulo p incongruen
Assim h, no mximo,
2
2
p - 1
tes mdulo p . Resta ser mostrado que os nmeros 12 ,22 ,...,
so in
2
p -1
congruentes mdulo p . Se m, n {1,2,...,
} so tais que m 2 n2 (mod p) ,
2
ento
m 2 - n2 0(mod p) ,
assim
temos
(m - n)(m + n) 0(mod p) ,
de

onde

conclumos

m + n 0(mod p)

ou

m - n 0(mod p) .

Uma

vez

Teoria dos nmeros

103

p -1
p -1
2p -2
e
, vale
0<n
0<m+n
= p -1
2
2
2
, isto , no h como m + n ser um mltiplo de p . Logo obtemos
que

0<m

m - n 0(mod p) e assim m n(mod p) . Porm, como m e n fazem par


te de um sistema completo de resduos mdulo p , conclumos que m = n .
2

p - 1
Assim, 12 ,22 ,...,
so nmeros incongruentes mdulo p , de onde obtemos o
2
resultado desejado.

A
7
T
1

EXEMPLO 4B:

Para p primo mpar, as nicas solues para a equao p2 1(mod p) so 1 e p - 1 .

EXEMPLO 5A:

A menos de congruncia mdulo 11, h exatamente 5 resduos quadrticos m


dulo 11.
EXEMPLO 5B:

A menos de congruncia mdulo 47, h exatamente 23 resduos quadrticos m


dulo 47.
No tpico seguinte, continuaremos analisando as equaes do tipo x 2 a(mod p) ,
para p primo e veremos como determinar se um nmero resduo quadrtico mdulo p .

104

Licenciatura em Matemtica

02

O SMBOLO
DE LEGENDRE

TPICO

OBJETIVOS

Apresentar o smbolo de Legendre


Relacionar o smbolo de Legendre com resduos
simples.

http://en.wikipedia.org/wiki/File:Legendre.jpg

matemtico francs Adrien-Marie Legendre (1752 - 1833)


contribuiu com estudos significativos na Estatstica, na
lgebra Abstrata, na soluo de Equaes Diferenciais e

na Teoria dos Nmeros. A seguir, definiremos o smbolo de Legen


dre, uma maneira simplificada de dizer se um nmero resduo qua
drtico mdulo p .
Definio 1: Dado o primo mpar p e o inteiro positivo a ,
Figura 1: Adrien-Marie Legendre

106

no mltiplo de p . O smbolo de Legendre de a por p , deno


a
tado por igual a 1, se a equao x 2 a(mod p) possui so
p

Licenciatura em Matemtica

luo, e vale 1, caso contrrio. Ou seja:


a 1, se a um resduo quadrtico mdulo p
=
.

p -1, se a no um resduo quadrtico mdulo p

EXEMPLO 1A:

Como visto no tpico anterior, 1 e 4 so resduos quadrticos mdulo 5, ento


1

5 = 1 e

A
7

5 = 1 . Uma vez que 2 e 3 no so resduos quadrticos mdulo 5, tem-se

T
2

2
3
= -1 e = -1 .
5
5
EXEMPLO 1B:

ATENO!
1 2 4
= = =1
7 7 7

Apesar da semelhana, no devemos


confundir o smbolo de Legendre com
uma frao simplesmente nem com o
nmero binomial.

A seguir, veremos algumas propriedades do smbolo

GUARDE BEM ISSO!

Vale
dizer
que
3 5 6
= = = -1
7
7
7

EXEMPLO 1C:

1
Para qualquer primo mpar p , vale = 1 .
p

de Legendre.

a
O smbolo de Legendre no
Proposio 1: Para qualquer primo mpar p , vale
p-1
a = 0 .


a =1 p

Demonstrao: Pelo que vimos no tpico anterior, me

definido se p|a , entretanto alguns


autores estendem a definio acima para

a
= 0 se p|a . Assim, por exemplo,
p
30
podemos considerar = 0 .
5

tade dos nmeros do conjunto {1,2,..., p - 1} so resduos


p-1
a
quadrticos mdulo p , de modo que na soma h

a =1 p
p -1
parcelas iguais a 1. Tambm sabemos que os demais nmeros, em igual quan
2
p -1
tidade, no so resduos quadrtico mdulo p , por isso h tambm
parcelas
2
iguais a 1. Somando todas as parcelas, obteremos soma zero, como desejado.
Adiante, provaremos um resultado tambm conhecido como critrio de Euler,

Teoria dos nmeros

107

que determina se um nmero resduo quadrtico apenas pela determinao do res


to de uma diviso.

Proposio 2: Se p um primo mpar e a um inteiro positivo no divisvel


p-1
a
por p , ento a 2 (mod p) .
p

A
7
T
2

a
Demonstrao: Se = 1 , ento existe um nmero inteiro x tal que x 2 a(mod p)
p
e como a 0 , claro que x no mltiplo de p , de onde obtemos que (x, p) = 1
. Usando o pequeno teorema de Fermat, sabemos que x p-1 1(mod p) . Nestes ter
mos, fazemos:

p-1
2

(x 2 )

p-1
2

(mod p) , que equivale a a

Juntando os fatos, conclumos que a

p-1
2

p-1
2

x p-1 (mod p) .

p-1
a
2
1(mod p) , ou seja, a (mod p) e o
p

primeiro caso fica provado. O segundo caso anlogo e vem do fato de que a quan
tidade de resduos quadrticos mdulo p

p -1
.
2

EXEMPLO 2A:
11-1
2
2

Vale 2 2 (mod11) , ou seja, 25 (mod11) . Como 25 = 32 , vale


11
11

2
25 -1(mod11) . Assim, = -1 e, logo, 2 no um resduo quadrtico mdulo 11.
11

EXEMPLO 2B:

3
Temos 36 (mod13) e, como 36 1(mod13) , vale que 3 um resduo qua
13
drtico mdulo 13.
Neste tpico, definimos o smbolo de Legendre, que uma maneira simplificada
de afirmar quando um nmero resduo quadrtico mdulo p e apresentamos o cri
trio de Euler, que transfere o problema de decidir se um nmero resduo quadr
tico para a determinao do resto da diviso entre dois inteiros. No tpico a seguir,
veremos outras propriedades do smbolo de Legendre.

108

Licenciatura em Matemtica

03
TPICO

LEI DA RECIPROCIDADE
QUADRTICA

OBJETIVO

Continuar o estudo dos resduos quadrticos,


analisando mtodos de simplificao do clculo do
smbolo de Legendre.

a
ontinuamos o estudo do smbolo de Legendre , analisaremos a existn
p
cia de solues para equaes do tipo x 2 a(mod p) . Para comear, vere
mos que o smbolo de Legendre, visto como funo de a , ou seja, com de

nominador fixo, completamente multiplicativa.


EXEMPLO 1A:

Proposio 3: Se a e b so inteiros positivos no divisveis pelo primo p , en


ab a b
to = . .
p p p
p-1
p-1
p-1
2

Demonstrao: Inicialmente, vemos que (ab) = a 2 .b 2 . Pelo critrio de


p-1
p-1
a
b
Euler (final do tpico 2), temos a 2 (mod p) e b 2 (mod p) . As
p
p
p
1
p
1
ab a b
a b
sim, . a 2 .b 2 (mod p) , isto , . (mod p) . Uma vez que
p p p
p p
o smbolo de Legendre assume apenas os valores 1 e 1, a congruncia acima
implica a igualdade desejada.

110

Licenciatura em Matemtica

11-1
3
Usando o critrio de Euler, podemos concluir que 3 2 (mod11) , ou seja,
11
3
5
5
3 (mod11) . Mas, como 3 = 243 deixa resto 1 na diviso por 11, temos que
11
3
2
= 1 . Pelo visto no exemplo 2a do tpico 2, vale = -1 . Usando a multipli
11
11
6 23
cabilidade do smbolo de Legendre, conclumos que = . (-1).1 = -1 ,
11 11 11
logo 6 no um resduo quadrtico mdulo 11.

EXEMPLO 1B:

A
7
T
3

an a
Para qualquer inteiro positivo a no divisvel por p , vale = . Dessa for
p p

a
ma, como os possveis valores de so 1 e 1, vale
p

a n
= 1 para qualquer n par.
p

EXEMPLO 1C:

25 1
Vale que 25 = 32 1(mod31) . Assim = = 1 , ou seja, 32 um resduo
31 31
5

2 32
quadrtico mdulo 31. Alm disso, = = 1 , de onde podemos concluir
31 31
2
que = 1 , isto , 2 um resduo quadrtico mdulo 31.
31

Como consequncia da multiplicabilidade do smbolo de Legendre, reduzimos o
trabalho de procurar resduos quadrticos a nmeros primos. Para comear, um cri
trio segundo os quais saberemos se 2 resduo quadrtico.
2 1, se p 1(mod8)
Proposio 4: Se p um primo mpar, ento =
.
p -1, se p 3(mod8)
EXEMPLO 2A:

2
Como 17 1(mod8) , ento = 1 , ou seja, 2 um resduo quadrtico mdulo 17.
17

EXEMPLO 2B:

2
Como 43 3(mod8) , ento = -1 , ou seja, 2 no um resduo quadrtico
43
mdulo 43.
A seguir, enunciaremos um resultado que simplifica o clculo do smbolo de Le
gendre, conhecido como a Lei da Reciprocidade Quadrtica.

Teoria dos nmeros

111

p-1 q-1
p q
.
Teorema: Se p e q so primos mpares distintos, ento . = (-1) 2 2 .
q p

EXEMPLO 3A:

41
Calcule .
43

A
7

Soluo:

43-1 41-1
.
41 43
Pela lei da reciprocidade quadrtica, temos . = (-1) 2 2 = (-1)21.20 = 1 .
43 41

T
3

41
43
A partir da, percebemos que e tm o mesmo sinal. Mas, se
43
41
43 2
43 2(mod 41) , logo = . Por fim, j que 41 1(mod8) , vale
41 41

2
= 1 . As 41

41 43 2
sim, = = = 1 , de onde obtemos que 41 um resduo quadrtico mdulo 43.
43 41 43
EXEMPLO 3B:

48
Calcule .
97
Soluo:

Como 48 no primo, comeamos pela fatorao 48 = 24.3 . Usando a mul


4
48 24 3 2 3
tiplicabilidade do smbolo de Legendre, = . = . . Como
97 97 97 97 97
o smbolo de Legendre assume apenas os valores 1 e 1, necessariamente tere
4
2
mos = 1 e o teste do resto na diviso por 8 desnecessrio. Passemos, en
97
3
to, determinao de . Para tanto, usemos a lei da reciprocidade quadrti
97
3-1 97-1
.
3 97
3 97
ca . = (-1) 2 2 = (-1)1.48 = 1 . Assim, e tm o mesmo sinal.
97 3
97 3
Uma vez que 97 1(mod3) , temos que 97 um resduo quadrtico mdulo 3, logo
41 43
97
= 1 . Concluindo: a partir da, obtemos que e tm o mesmo sinal.
3
43 41
48
Mas 43 2(mod 41) , logo = 1.
97

EXEMPLO 4:

Mostre que no existe inteiro n tal que 7|4n2 - 3 .

112

Licenciatura em Matemtica

Soluo:

Observe que 7|4n2 - 3 equivale a 4n2 3(mod7) , ou seja, (2n)2 3(mod7) e, se


fizermos x = 2n , teremos x 2 3(mod7) . Podemos aqui usar o que foi feito no tpico
1, no qual obtemos que 3 no um resduo quadrtico mdulo 7, para afirmar que a
equao no possui soluo. Alternativamente, usando a lei da reciprocidade quadrti
3-1 7-1
.
3 7
3 7
ca, podemos ver que . = (-1) 2 2 = (-1)1.3 = -1 , isto , e tm sinais
7 3
7 3
7
3
contrrios. Mas, como 7 1(mod3) , temos = 1 , de onde obtemos que = -1 e,
3
7
da mesma forma, conclumos que 3 no um resduo quadrtico mdulo 7.

A
7
T
3

Com estes dois testes simples, encerramos nossa aula sobre resduos quadrti
cos. Vimos como a investigao sobre existncia de solues para equaes do tipo
x 2 a(mod p) pode ser bem simplificada.

Teoria dos nmeros

113

Aula 8

Problemas diversos
Caro(a) aluno(a),

Chegamos nossa ltima aula de Teoria dos Nmeros. Aqui revisitaremos os principais resultados apresentados no decorrer do curso, atravs de problema resolvidos
de diversos nveis de dificuldade. No hesite em procurar nas aulas passadas as
definies pertinentes a cada situao. Fique atento tambm s hipteses de cada
enunciado e aos momentos nos quais elas so usadas.

Objetivo:
Apresentar problemas resolvidos sobre a teoria desenvolvida nas aulas anteriores e discutir
a aplicao das tcnicas apresentadas em suas solues.

115

01

MISCELNEA
DE EXERCCIOS

TPICO

OBJETIVO

PROBLEMA 1:

Mostre que a soma de dois nmeros inteiros mpar se, e somente se, um deles
for par e outro for mpar.
Soluo:

Inicialmente, verifique que, se a for par e b for mpar, podemos escrever a = 2m e


b = 2n + 1 para certos inteiros m e n . Da teremos a + b = 2m + 2n + 1 = 2(m + n) + 1 ,
que um nmero mpar, isto , a soma de um nmero par com um nmero m
par resulta em um nmero mpar. Se considerarmos a e b ambos pares, pode
mos escrever a = 2m e b = 2n para certos inteiros m e n . Assim encontraremos
a + b = 2m + 2n = 2(m + n) , que um nmero par. Se a e b forem ambos mpares,
podemos escrever a = 2m + 1 e b = 2n + 1 para certos inteiros m e n . Da teremos
a + b = 2m + 1 + 2n + 1 = 2m + 2n + 2 = 2(m + n + 1) , que um nmero par. Dessa
forma, a soma de dois nmeros pares sempre par e a soma de dois nmeros mpares
tambm par. O mesmo resultado vale para a diferena de dois nmeros.
PROBLEMA 2:

Mostre que se a e b so nmeros mpares, ento a 2 + b2 no pode ser um qua


drado perfeito.

116

Licenciatura em Matemtica

Soluo:

Comecemos observando que, se a e b so mpares, o mesmo ocorre com a 2 e b2 ,


logo a 2 + b2 um nmero par. Para que um nmero par seja um quadrado perfei
to, necessrio que ele seja um mltiplo de 4, pois 2|n2 2|n 4|n2 . Mas, co
mo a mpar, vale a 1(mod 4) ou a 3(mod 4) e, em ambos os casos, tem-se
a 2 1(mod 4) , o mesmo ocorrendo para b , isto , vale b2 1(mod 4) . Somando as

A
8

duas congruncias, obtemos a 2 + b2 2(mod 4) , de onde conclumos que a 2 + b2


no um mltiplo de 4 e, assim, no pode ser um quadrado perfeito (como conse
quncia deste resultado, um tringulo retngulo com lados de medidas inteiras tem

T
1

pelo menos um dos catetos de medida par).


PROBLEMA 3:

Prove que M = 32n+1 + 2n+2 um mltiplo de 7 para qualquer inteiro positivo n .


Soluo:

Observe inicialmente que 32 2(mod7) , logo

(32 )

2n (mod7) e, assim,

32n+1 2n.3(mod7) . Da segunda parcela da soma que define M , podemos afirmar


2n+2 = 2n.22 = 2n.4 que gera a congruncia imediata 2n+2 2n.4(mod7) . Somando
ento as duas ltimas congruncias, obtemos 32n+1 + 2n+2 2n.3 + 2n.4(mod7) , mas
2n.3 + 2n.4 = 2n.(3 + 4) = 2n.7 , que , claramente, um mltiplo de 7.
PROBLEMA 4:

Mostre que a representao decimal de um nmero quadrado perfeito no pode


ter algarismo das unidades terminando em 2, 3, 7 ou 8.
Soluo:

O algarismo das unidades na representao decimal o resto da diviso do nmero


por 10. Considere, ento, os possveis restos e observe o que acontece com o quadrado:

n 0(mod10) n2 0(mod10)

n 1(mod10) n2 1(mod10)

n 2(mod10) n2 4(mod10)

n 3(mod10) n2 9(mod10)

n 4(mod10) n2 6(mod10)

n 5(mod10) n2 5(mod10)

n 6(mod10) n2 6(mod10)

n 7(mod10) n2 9(mod10)

n 8(mod10) n2 4(mod10)

n 9(mod10) n2 1(mod10)

Assim, os possveis algarismos das unidades de um quadrado perfeito so apenas

Teoria dos nmeros

117

0, 1, 4, 5, 6 e 9. De modo que, se um nmero terminar em algarismo diferente des


tes, quando escrito na base 10, ele certamente no ser um quadrado perfeito. Como
consequncia deste critrio, podemos, sem nenhum clculo, auxiliar, dizer que os
nmeros

10253443 ,

3278812 e

20003417 so irracionais, pois os radicandos

no so quadrados perfeitos.
PROBLEMA 5:

A
8

Prove que o produto de trs nmeros inteiros consecutivos sempre divisvel por 6.
Soluo:

T
1

Em uma sequncia de trs inteiros consecutivos, h necessariamente um mltiplo


de 3, de modo que o produto de trs inteiros consecutivos um mltiplo de 3. De
maneira anloga, certamente um dos fatores ser um nmero par e, assim, mltiplo
de 2. Assim, o nmero obtido divisvel por 2 e por 3 ao mesmo tempo, sendo por
tanto, mltiplo de 6.
PROBLEMA 6:

Usando o algoritmo de Euclides, determine o mximo divisor comum entre 432 e


28, em seguida determine o mnimo mltiplo comum entre 432 e 28.
Soluo:

Comeando pela diviso de 432 por 28, obtemos


432 = 28.15 + 12
Assim, o problema transferido para a determinao do mximo divisor comum
entre 28 e 12. Dividindo 28 por 12, obtemos
28 = 12.2 + 4
Desse modo, temos que o mximo divisor comum entre 28 e 12 o mesmo que
entre 12 e 4, mas, como 4 um divisor de 12, temos (28,12)=4. Logo, o mximo divi
sor comum entre 432 e 28 4. Usando a identidade [a, b ].(a, b) = a.b , ganhamos que
432.28
432.28
[432,28] =
=
= 432.7 = 3024 , ou seja, o mnimo mltiplo comum
(432,28)
4
entre 432 e 28 3024.
PROBLEMA 7:

Mostre que a equao 42x + 180 y = 14 no possui solues inteiras.


Soluo:

Do estudo de equaes diofantinas, sabemos que ax + by = c possui solu


o (x, y ) se, e somente se, (a, b)|c . Para a equao dada, basta ver que
(42,180) = 6 , mas 6 no divide 14.

118

Licenciatura em Matemtica

PROBLEMA 8:

Determine o resto na diviso de 22.33.44.55 por 7.


Soluo:

Como 22 1(mod7) , 33 5(mod7) , 44 2(mod7) e 55 6(mod7) , temos, pela


multiplicao das congruncias, que 22.33.44.55 1.5.2.6(mod7) . Como 1.5.2.6=50
deixa resto 4 na diviso por 7, temos que 22.33.44.55 deixa resto 4 na diviso por 7.

A
8

PROBLEMA 9:

Determine o nmero inteiro positivo n que satisfaz 12|n e t (n) = 14 .

T
1

Soluo:

Seja

n = p1a1 . p2a2 ..... pkak

fatorao

em

primos

de

n,

temos

que

t (n) = (a1 + 1)(a2 + 1)...(ak + 1) . Assim, devemos procurar as formas segundo as


quais 14 pode ser escrito como produto de nmeros inteiros positivos. So elas
14 = 14.1 , e neste caso, teremos n = p13 , para algum primo p , ou 14 = 7.2 , caso no
qual vale n = p6q para primos p e q . Como 12|n e 12 = 22.3 , necessariamente os
primos 2 e 3 devem aparecer na fatorao de n , de modo que a primeira opo no
ocorre. Ento temos n = 26.3 ou n = 36.2 . Mas a segunda opo no ocorre, pois o
expoente do 2 deve ser no mnimo 2 para que se tenha 12|n . Assim, a nica possi
bilidade n = 26.3 = 192 .
PROBLEMA 10:

Encontre todos os restos possveis na diviso de um quadrado perfeito por 8.


Soluo:

Uma vez que os possveis restos na diviso por 8 so apenas 0, 1, 2, ..., 7, avalia
remos apenas os possveis restos na diviso de n2 por 8 nos seguintes casos:
se n 0(mod8) , temos n2 0(mod8) e o resto , portanto, 0
se n 1(mod8) , temos n2 1(mod8) e o resto , portanto, 1
se n 2(mod8) , temos n2 4(mod8) e o resto , portanto, 4
se n 3(mod8) , temos n2 9(mod8) e o resto , portanto, 1
se n 4(mod8) , temos n2 16(mod8) e o resto , portanto, 0
Esgotadas todas as possibilidades, conclumos que um quadrado perfeito deixa
resto 0, 1 ou 4 na diviso por 8.
PROBLEMA 11:

Mostre que, se k 7(mod8) , ento k no pode ser escrito como soma de trs
quadrados perfeitos.

Teoria dos nmeros

119

Soluo:

Pelo problema anterior, um quadrado perfeito deixa resto 0, 1 ou 4 na diviso por


8, logo a soma de trs quadrados ser congruente soma de trs nmeros do conjun
to {0,1,4} . Porm 7 no pode ser escrito como soma de trs dos nmeros dados, por
isso impossvel que um nmero que deixa resto 7 na diviso por 8 possa ser escrito
como soma de trs quadrados perfeitos.

A
8

PROBLEMA 12:

T
1

Soluo:

Determine o resto da diviso de 196 + 4424 por 7.


Como 19 no divisvel por 7, temos, pelo Teorema de Fermat, que 197-1 1(mod7) ,
ou seja, 196 1(mod7) . Usando argumento semelhante, conclumos que
446 1(mod7) , de onde obtemos (446 ) 14 (mod7) , isto , 4424 1(mod7) . Por
4

fim, temos 196 + 4424 1 + 1(mod7) e o resto procurado , portanto, 2.


PROBLEMA 13:

Encontre a quantidade de nmeros inteiros positivos menores que 3600 que so


mltiplos de 2, 3 ou 5.
Soluo:

Observe que a fatorao de 3600 em primos 3600 = 24.32.52 . Assim os fato


res primos de 3600 so exatamente 2, 3 e 5, de modo que os mltiplos de 2, 3
ou 5 so aqueles que no so relativamente primos com 3600. Podemos encontrar
a quantidade dos que so relativamente primos com 3600 pela funo de Euler
1 1 1
1 2 4
f(3600) = 3600.1 - .1 - .1 - = 3600. . . = 960 . Assim, se retirarmos
2 3 5
2 3 5
esses 960 nmeros dos 3599 inteiros positivos menores que 3600, obteremos os 2639
que so mltiplos de 2, 3 ou 5.
PROBLEMA 14:

Encontre o menor inteiro positivo n para o qual t (n) = 13 .


Soluo:

Seja

n = p1a1 . p2a2 ..... pkak

fatorao

em

primos

de

n,

temos

que

t (n) = (a1 + 1)(a2 + 1)...(ak + 1) = 13 , porm, como 13 um nmero primo, no po


der ser escrito como produto de dois nmeros menores que ele. Logo os nmeros
que possuem exatamente 13 divisores positivos so da forma n = p12 , com p pri
mo. De modo a minimizar o valor de n , consideramos o menor primo e obtemos
n = 212 = 4096 .

120

Licenciatura em Matemtica

PROBLEMA 15:

Prove que se n no um quadrado perfeito, ento t (n) par.


Soluo:

Se n no um quadrado perfeito, ento pelo menos um dos expoentes de sua


fatorao em primos mpar, de modo que pelo menos um dos consecutivos destes
expoentes par. Mas, quando

um dos fatores par, o produto par, logo a quantida

A
8

de de divisores de n que calculada pelo produto dos consecutivos dos expoentes


de sua fatorao em primo ser par.

T
1

PROBLEMA 16:

Determine todos os nmeros inteiros positivos menores que 12 que possuem in


verso mdulo 12.
Soluo:

Um nmero n possui inverso mdulo 12 quando existe um inteiro k tal que


nk 1(mod12) e isso ocorre se, e somente se, (n,12) = 1 . Investigando quais dos in
teiros positivos menores que 12 lhe so relativamente primos, obtemos a lista 1, 5,
7, 11. Mais ainda, cada um dos nmeros listados seu prprio inverso mdulo 12.
PROBLEMA 17:

Mostre que a equao n3 + 5n - 27 = 0 no possui soluo inteira.


Soluo:

Veja que a equao equivalente a n3 + 5n = 27 , ou seja, n(n2 + 5) = 27 . Obser


ve que n2 + 5 um nmero necessariamente positivo e maior que n , de modo que
escrevemos 27 como produto de dois nmeros inteiros positivos, mas s h duas ma
neiras de fazer isso: 27 = 1.27 , caso em que n = 1 e n2 + 5 = 27 , o que no se ve
rifica; e 27 = 3.9 , caso em que n = 3 e n2 + 5 = 9 , o que igualmente falso. Assim
conclumos que a equao dada no possui razes inteiras.
PROBLEMA 18:

Dado o nmero inteiro B > 3 , mostre que o nmero (1331)B um cubo perfeito.
Soluo:

Aqui basta ver que (1331)B = B 3 + 3B 2 + 3B + 1 = (B + 1)3 , que o cubo do in


teiro B + 1 .
PROBLEMA 19:

Mostre que existem infinitos nmeros inteiros n para os quais 10|f(n) .

Teoria dos nmeros

121

Soluo:

Observe que f(11) = 10 , logo, se m for relativamente primo com 11, teremos
f(11m) = f(11)f(m) = 10f(m) , assim 10|f(11m) , isto , 10|f(n) para todo inteiro
positivo da forma n = 11m , com (11, m ) = 1 . Mas h infinitos nmeros que podem
ocupar o lugar de m , por exemplo todos os outros primos.
PROBLEMA 20:

A
8

Mostre que, para qualquer inteiro positivo n > 1 , existem infinitos nmeros inteiros
m para os quais t (m) = n .

T
1

Soluo:

Observe que t (m) = n ocorre para qualquer nmero da forma m = pn-1 , onde p
primo. O resultado segue do fato de que h infinitos nmeros primos.
PROBLEMA 21:

Determine o algarismo das unidades de 7202 , quando expresso em base decimal.


Soluo:

O algarismo das unidades de um nmero o resto da diviso pela base. As


sim, devemos determinar o resto da diviso de 7202 por 10. Sabemos pelo teorema
de Euler, que a f ( n ) 1(mod n) , para quaisquer inteiros positivos a, n relativamen
te primos. Assim, temos 7f (10) 1(mod10) . Mas f(10) = 4 , da vale 74 1(mod10) .
Dividindo 202 por 4, obtemos quociente 50 e resto 2, de onde conclumos
7202 = 750.4+2 = (74 ) .72 150.72 (mod10) . Por fim, como 72 = 49 9(mod10) , o al
50

garismo procurado 9.

122

Licenciatura em Matemtica

Referncias
LANDAU, Edmund. Elementare Zahlentheorie. Leipzig: S. Hirzyel Verlag, 1927.,
SANTOS, Jos Plnio de Oliveira. Introduo Teoria dos Nmeros. Rio de Janeiro: Instituto de
Matemtica Pura e Aplicada, 1998.

CURRCULO
JNIO KLO DE CASTRO SOUSA

Jnio Klo comeou seus estudos de Matemtica em 2000, quando ingressou no bacharelado da
Universidade Federal do Cear, colando grau em julho de 2004. A partir de 2001 e por trs anos, foi
monitor de Clculo Diferencial e Integral na UFC, desempenhando atividade de acompanhamento e
tira-dvidas para alunos de graduao. Durante os anos de 2006, 2007 e 2008, foi professor da UFC,
com turmas de diversos cursos, ministrando aulas de lgebra Linear, Equaes Diferenciais, Vari
veis Complexas e Geometria Hiperblica, entre outras. Desde o comeo de 2009 professor do Insti
tuto Federal de Educao, Cincia e Tecnologia do Cear, atuando nos campus de Fortaleza e Mara
cana, nos cursos presenciais e semipresenciais.

Potrebbero piacerti anche